Tải bản đầy đủ (.pdf) (26 trang)

Giải nhanh các dạng bài tập trắc nghiệm anh văn p3

Bạn đang xem bản rút gọn của tài liệu. Xem và tải ngay bản đầy đủ của tài liệu tại đây (22.22 MB, 26 trang )

vyiy i i M n n i v n i r u w F f Rnang vi?i

+

V 6 mat rniQ imhTa: "Ccim phien ban tm lai ctien ihogi giup

+

Ve mat ngCr phap: Ta c6 can true: Would you mind + V - i n g . . . ?

Cam phien (anit, chi.• •) I"'"

i''

(^"'''

toiT

+
+

Ve mat ngu nghTa: " Peter hoi lice vi dd khong hoi vay Mary 30 do la."
Ve mat ngu' phap: Dong tir Regret c6 2 dang thirc:

"fi^'l '1^'' "'.')

Regret + V-ing: lay lam tiSc vi dd Idm gi

Cac dap an con lai khong phu hgp dang thuc sau Would you mind?

Regret + V to-inf: lay lam tiec khiphai lam gi



Cau 24: Dap an A - who
+

Ve mat ngir ngliTa: ''Bill

Microsoft
+

Company, la mot

Gates, chit tich sang lap tap doan phdn

dgc biet la khi thong hdo mot thong tin hoac su viec khong tot.

mem

L u u y: sau Regret ta thuong diing hinh thuc V to-inf d6i vai cac dong

tiphu."

tir inform, tell.

Ve mat ngu phap: Quan sat cau de ta thay: day la menh de quan he khong

Loai A - to ask- v i khong diing ngu nghTa.

H.

day, chi c6 who la dap an dung v i who dung de thay the cho nguoi va lam chuc


Loai C- asked- v i khong dung ngu phap.

'

nang chu ngu'

Loai D- ask- v i khong dung ngti' phap.

xac djnh (loai menh de co dau phSy). Do do, ta khong dung phuoiig an D {that). 6

Cac dap an con hii khong phu hop vi:
nang tan ngfr
C - whose: dai t u quan he dung de thay the cho cac tinh tir sa hiJu.
D - that: dai tir quan he that khong diing trong m$nh de quan he khong xac djnh
Cau 25: Dap an B - So do I
+

Ve mat ngCr ngliTa: "Toiyeii

+

Vk mat ngu phap: Ta can nha cau triic sau:

thich am nhgc Idng man."

+ Ve mat ngir nghTa: ''Chuyen bay kh&i hanh den thdnhpho
hoan lai do suong mu day dgc."

+ Ve mat ngu nghTa: " Do Id mot ngay qua Ignh lea den ndi toi phai

chiec do Ignh miia dong."

Do, does +S

t r i i c trcMi

cong

voi

cau de dang dimg dong

tii-

thuong va a dang

Cac dap an con hjii khong phu hop vi:
B- so that + Clause :de, de ma- la lien tir ma dAu mot menh de phu chi muc dich.

Cau 26: Dap an C - had met
Ve mat ngu nghTa: "Zc~ ra toi nen gap hem vao nam triivc."

If

Ve mat ngu phap: Ta bict I f only

o n l y CO

chinh


C- and : va- la lien tir diing de noi hai y cua mot cau hay hai menh de.

hoac "7>5/ tiec da

khong gap ban vao nam trm'rc"
+

lii each dien dat khac cua Wish, sau

D - i n order to + V ( bare inf): de, cot de
Cau 31: Dap an D - perfect^perfectly
-Ve mat ngu nghTa: " Toi rat vui khi bdo cho ban biet rdng bay gia may thu

2 dang sau:

hogt dong mot each tuyet vai. "

Dan<; 1: I f only + Q K D - hoac Were cho tat ca cac ngoi d^ dien ta mgt dieu mong
^

niuoii khong c6 that a hien tai.
Dang 2: If only+ Q K I I T - de dien ta mot dieu mong muon khong c6 that a qua khir.
Vay can cu vao dang cau va cau de c6 chira tr^ng ngir chi thai gian "-last year"
{nam

khodc

.... such + (a/an) noun + t h a t . . . . q u a .... den noi

kh;ing djnh, B- So do I la dap an dimg nhat.

+

,|.

+ Ve mat ngu phap: Ta c6 cau triic:

Do/ docs
cau

dd bi

+ Ta xet thay: Cac phuong an B, C va D khong chia dong tii- a dang bj dong.
Xet chi CO phuong an A la dung- chuy^n bay t y no khong th^ tri hoan ma phai
dupe tri hoan.

S + is/ am/ arc, + too
tiieo

Manchester

Cau 30: Dap an A - open

So/ Neither + is, arc, am +S

Chieu

^ V'

Cau 29: Dap an A - was delayed


B - whom: dai tir quan he dung d l thay the cho cac danh t u chi nguoi lam chuc

ngoai) thi C la phuong an diing.

Cau 27: Dap an C - do my parents

work

+

Ve mat ngir nghTa: " Cha

toi khong con lao dong cue nhoc moi ngciy nica"

+

Ve mat ngu phap: Cau nay c6 no longer dirng dau cau nen ta phai dao ngir.

Ve mat ngii' phap: a cau nay can mot trang tir de b6 nghTa cho dong tir . V i

^^y, tinh tir perfect{

hoan hdo, tuyet vai) phai ddi thanh trang tir perfectly

{mot

^och hoan hdo)
32: Dap an C - possible—>possibly
+ Ve mat ngir nghTa: " Neu ban tim thay sach cua toi, vui long dua no lai giup toi."
+ Ve mat ngu phap: a day, ta khong the diing mgt tinh tir " possible" dk bo


Vay C la dap an diing v i c6 dua t r g dong tir ra ngay sau no longer. Cac phuang an

•^Shia cho mot dong tir " return". V i vay, ta phai chuyen no sang mot trang tir la "
Possibly".
'


con lai khong diing ve mat ngu phap cua cau.

^'^K 33: Dap an A - o i w i n

;

Cau 28: Dap an B - asking
104

!65


+ Ve mat ngu' nghTa: " Vao sang hom sau anh ta thuc day va di bp xuong soiij^

C- Day la ngoi nha d?p nhSt ma tru-oc do toi chira bao gia thay.

de tarn rua ban than."
+

Cau nay sai ve mat nghTa.

Ve mat ngCr phap: cau van sai ve each dung giai tir.


D- Do la ngoi nha dep nhat ma toi chua bao gia nhin thay.

C a u 34: Dap an D - q u i c k ^ q u i 6 k l y
+

V ^ mat ngu' nghTa: " Chung toi dupe mong dgi tra lai tung cau hoi mot cneh

nhanh ehong."
+

Cau nay hop ve mat ngir nghTa nhung khong hgp ve mat thi so v a i cau de.

Ve mat ngir phap: theo sau dong tir phai la mot trang tir de bo nghTa cho cjiu

Doi quick (adj): nhanh, nhanh chong thanh quickly ( adv): mot each nhanh chon^,
C a u 35: Dap an A - w i l l r a i n ^ r a i n s

Cau nay sai ve mat nghTa.
Cau 38:
+ C a u goc mang nghTa: " Toi da mat 5 g i a de sira mai nha nay."
+ Dap an diing: A - / had to spend 5 hours to repair this roof. (Toiphai
giff de si'ra mai nha nay.)

Vh mat ngfr nghTa: " Neu troi mua, toi se den gap ban d trong xe."

+

Ve mat ngir phap: Day la cau dieu kien loai I, dien ta mot dieu eo kha nang


It takes/ took + somebody+khoang thoi gian+ V(-to inO+

xay a hi?n tai hoae tuong lai voi eau true:

= Spend one's time in doing something/ something
Ai do bo r a / ticu ton bao nhieu thoi gian vao vice gi/ lam viec gi.

If + clause ( H T D ) , main clause ( T L D )
C a u 36:

+

C a u goc mang nghTa: " Me toi da khong goi dien cho toi cho den khi ba ay

tra ve tir thanh pho."

Phuong an sai:

B- M a i nha nay diroc sua trong 5 gia dong ho.
* Cau nay sai vi trong cau c6: F o r + khoang thoi gian: khoang bao nhieu thoi

+ Dap an dung: B- Not until coming back to the city did my mother phone me.
(MQ toi da khong goi dien cho toi cho mai den khi ba ay tra \h tir thanh ph6;
Dita veto ngu nghTa ciia cau thi D la dap an duy nJidt diing.
.'l"'

+ Phuong an sai:
A - K h i me toi tra ve thanh pho, ba ay nen dien cho toi.

gian....

Do do, cau nay khong dung ve mat ngu phap.
C - Toi da sua mai nha nay trong 5 gia dong ho. Cau nay su dyng thg sai khien:
Sl(chi nguoi)+ have/had +S2(chi vat)+ V3/ed...
Cau nay dung ve each dung nhung have trong cau dugc dung a hien tai, khong
hgp thi v a i cau de a qua khu.

Cau van nay sai ve mat nghTa vi c6 them tir should.


mat 5

Ta C O hai cau true sau tirong dirong ve mat nghTa:

+

+

" <
<

C- Me toi khong dien cho toi cho mai tai khi ba khong tra ve thanh pho.
Cau nay sai hoan toan ve m$t nghTa so vai cau de.

D- Toi khong the sua ngoi nha nay ehua tai 5 gia d6ng ho. Cau nay sai ve mat nghTa.
Cau 39:

D- Ngay khi m? toi vira tra ve thanh pho thi ba ay dien cho t o i .

+ C a u goc mang nghia: " Phong b^p khong Ion nhu phong khach."


Cau nay cung sai ve mat nghTa so vai cau de.

Nhin vao cau de , ta biet day la so sanh b5ng a dang phiT djnh, va dang cau nay
se tirang duong vai dang cau so sanh hon nhu sau:

C a u 37:
+

C a u goc mang ngliia: " T o i chira bao gia thay mot ngoi nha nao d^p ho"

ngoi nha do."

81 + V + notas/so + a d j / a d v + a s +
=S2 + V + short adj/adv +er

+ Dap an diing: B- It was the most house 1 had ever seen . (Do Id ngoi nho
dep nhdt md toi da timg

thdy.)

+ Dap an diing: C- The living room is larger than the kitchen.

do hi Idn dtni tien

huiigi...)

Ma theo de ra dong tiJ cua cau dang dugc dung a qua k h u nen dong tir a cai'
viet lai cung phai chia a qua khu do do:
had never seen ....=It was the most + adj +N+ Claiise( I had ever seen)
+ Phuffng an sai:

A - Day la ngoi nha dep nhat ma toi tirng thay.
106

+ than+Sl

more +long adj/adv

... have never + V3/ed.... = It is + so sdnh /that cua tinh tir +N+ clause ( S*
tinig bao gi&laiugi

S2

Doi chieu v a i eau true tren thi C la dap an dung.

Ta CO 2 cau true sau tuang duang ve mat nghTa:
have ever+ V3/ed...chira

^*

{Phong khcich Ian hom nha hip.)
+ Phuong an sai:

••"1 rU'\\i\

.|^-Ph6ng khach Ion nhu phong bep. Cau nay diing ve each diing nhung sai ve
nghTa so v a i eau de.
^ - Phong khach khong I O M I nhu phong b i p . Cau nay diing ve each diing nhung
ye mat nghTa so voi eau de.



oiy

D- Phong khach nho hon phong bep. Cau nay diing ve each dung nhung sai v\
mat nghTa so voi cau de.
Cau 40:
+ Cau goc mang nghTa: " H o khong con lam viec tai cong ty nua."

most picturesque scenes."
,

A - " S u dung cac phuong tien giao thong."

V j tri thu- nhat: dung truac mot dpng tir thuong, luc nay no c6 chuc nSng nlur

C - " D i den cac rap hat

,,

D- "Dgc sach vao luc ranh r o i . "

A - Ho khong con khong lam vice tai cong ty nua.

A/

; s,:

'I'fW?

Cau nay sai ve mat nghTa so voi cau goc va mot cau khong the viia diing the phu
djnh va no longer.


^

^

j^;,, ^^^^ r o ;

Cau 43: Theo doan van, cau nao duoi day la khong diing:

+ Phu-ong an sai:

I Tviiciiiy v i ^ i

• '-'

Cac chon lu-a khac sai y :

N h u ta biet: no longer: khong con nira c6 hai v i t r i dung trong cau.

V j t r i thi'r hai: se xay ra hien tugng dao ngir neu no longer dung dau cau.

u vvi

Dien giai: " Xem cac bg p h i m . " Y nay dugc de cap trong do^n van tir cau "

I f one evening

lam vi4c tai cong ty nua.)

mot trang tir.


ivi I V

Q^n 42: Theo doan van, chiing ta c6 the di den nhung hang ciing ngo hem cua
trai dat thong qua:
- Dap an: B
.
?
,

+ Dap an diing: C- No longer do they work at the company. (Ho khong co,,

II'inii

:iw

^

..fmi

a

-

Dap an: C

.

Dien giai: " N g u d i dan song d thanh pho khong thich dgc sach." Y" nay hoan


toan khong dugc de cap den trong doan van.
Cac chon lira khac sai vi:
A - " D i den cac rap hat khong phai la mot hinh thuc giai t r i c6 ve tre con.": Y nay

B- Cau nay sai v i mpt cau khong the vira dung no longer va any longer.

dugc de cap vao dong cuoi cimg cua doan van: " I f that pastime

D- Cau nay sai v i mot cau khong the viia c6 no longer va any more.
Huong dan giai tu' cau 4 1 - 4 5

actors or actress."

B- "Phim anh dua ta den nhung nai each xa chiing ta a . " Y nay dugc de cap den

Bai djch de nghj:
Ci thanh pho, moi thu diiong nlur duoc tao ra de dein den cho ngiioi dan thaiili

llii

sir thoai mai va thuan tien nhat c6 the. Ban thi'ch di tir mot dja diem nay den mot mot
dja diem khac u? Ban c6 the tiiy thich chon v6 s6 cac loai phuong tien giao thong voi
toe dQ khac nhau chSng han nlur: cac loai xe buyt, xe dien, tdc xi hay di lai bang \

a cau 6 cua doan van: " w i l l transport you to....the globe"
D- " Thanh pho dem l ^ i cho chung ta nhieu hinh thuc giai t r i khac nhau." : Y
nay dugc de cap den trong doan van tir " I f one

actors or actresses.'^g


Cau 44: Tir doan van, chiing ta biet rang:
-

:.^\mM.
tslHI •

rAqifepisiS,

Dap an: D

nhay mat c6 th^ dua ban den nhung noi xa xoi heo lanh nhat cua qua dja cau nay \

- Dien giai: " K h i chung ta khong muon dgc sach, chung ta c6 the di den rap
hat." Y nay dugc de cap den d trong doan sau: " you do not feel inclined to read,
you may go to moving pictures"

hifn ra truoc mk chung ta nhung canh tuong tliien nhien day ky vT . Va neu tro gun

Cac chon lu-a khac sai y rjb-nMil &m"^^

dien ngSm
khong

CO

Neu bong choc mot dem nao do ban khong biet phai lam gi va ban

hirng thu d^ doc sach


tlii

ban c6 the di xeni phim ma chinh dieu nay

trong

tri nay c6 ve hoi tre con doi voi ban thi ban c6 ihk den cac buoi bieu dien ca nhac va

(f'-m-i^

A - "Tac gia thich dgc sach hon la xem p h i m . "

>

thirong thiic giong hat ciia nam va nir ca sT noi tieng nhat a do bieu dien.

B - " C h u n g ta nen di xem phim neu nhu chung ta CO du thai gian."

Cau 41: Chon tua de diing nhat cho doan van.

C- " K h o n g CO gi de chju hon viec di xem p h i m . "

.'

gtr6:,r'.
i.

-

Dap an: B


Cau 45: " 6 thanh pho, moi thu duong nhu- du-oc tao ra d6 dem lai su- thosii

-

Dien giai: " N h u n g Igi ich cua cuoc song is thanh t h j . " N o i dung toan bai J^'

"•^i va thuan tien cho ngu-oi dan pho thj." Dira vao y cau tren, dieu tac gia

cap d i n nhung thuan Igi khi song a thanh pho.
Cac chon lua khac sai vi:

" Dap an: B

A - "Cac phuong tien giao thong a thanh ph6.": Y nay chi dugc de cap nhu
mot phan ciia doan van tir " D o you wish....railways, e t c . . . "

^ - Dien giai: " Co nhieu c o so hien dai dugc tao ra cho nguai dan a nai day."
,

C- "Cac hinh thirc giai t r i a thanh pho.": Y nay chi dugc de cap nhu la mot pi'-'"
trong doan van tir " I f one

actress"

D- " L a m sao d l song a thanh pho.": Y nay hoan toan khong dugc de cap
trong doan van.
108

, j


'•'at sy muon noi den la:

91 dung toan bai de cap den nhung thuan Igi khi song a thanh pho.

chon lya chua neu duoc y khsii quat cua tac gia:
^ - " N g u o i dan thanh pho thich sy thoai m a i . "

, ,.

"Cuoc song a thanh pho lam cho mgi thu tro nen thoai mai h o n "
^ - "Nhieu thir co ve duang nhu thoai mai bai v i chiing co a thanh pho."

.j
109


Cty TNHH MTV DVVH Khang Vi?t

G\&\h CDBT TN Anh VSn - NgO VSn Minh

Huong dan giai tir cau 46 - 50
Doc kidoan

van sau va digit phumtg an cfiing (ling v&i A hoac B, C, D) ch,,

Dap an C: name (v): dat ten
Dap an D: know(v): biet, nhan tlu'rc dugc


cau 49:

mSi cho trong tie 46 din 50.
Bai dich dc nghj:
Lenin sinh vao diang t u nam 1870 a Simbrish. Cha ong qua dai klii ong

.

Dap an: D - published

-

Dien giai: publish (v) xuat ban, phat hanh ( sach, bao c h i . . . . )

tuoi.Vao thang 9 nam 1887, ong theo hpc truong Dai hoc Kazan va ong da bi bfn

Cac dap an con lai khong hop ve nghTa can dien trong cho trong.

do tham gia vao cac phong trao phan doi cua sinh vien va bj duoi hoc. Sau do, 6iig

Dap an A : distribute(v): phan phfii, phan phat

bat dau nghien cuu ve cac tac pham van hoc d cac the he truoc cua nen each nian„

Dap an B: print (v): viec in an, cho in (sach, bao c h i . . . . )

X 6 Viet va ve cac dao luat. Ong da c6 duoc giay phep hanh nghe luat vao nam

Dap an C: announce(v): truyen bo, cong b6


1892. MQt nam sau do, ong bat dau quan tam vao chu nghTa cua Mac. Vao thang 4

cau 50:

nam 1899, ong cho ra doi quyen sach dau tay mang tira de "Sir phat trien cua chu

-

Dap an: B - devote

nghTa t u ban a X 6 Viet". Vao thang mot nam 1900, ong da dSn chau A u . Cung nam

-

Dien giai:

do vao thang 12, ong da cho ra doi to bao Iskra. Ong da chuyen den song a nu6c
Anh vao thang 4 nam 1903. K h i ong den London, ong rat nga ngang khi chung kicn
sir phan biet giau ngheo ro r f t cua nguoi London. Ong da cong hien tron doi miiili de
dan dat sir nghiep each mang cua giai cap v6 san X 6 Viet di den thang loi lioan loan,
Vao ngay 2 i tliang 1 nam 1924, ong da qua dai do chiing dQt quy.
C a u 46:

'•'ml

Ta CO cum tu: devote one's life to something/ doing something: cong hien,
danh tron cuoc doi cho mot viec gi/ lam vice gi.
Cac dap an con lai khong phu hop ve each dung nghTa can dikn trong cho trong.
Dap an A : bet(v): danh ca, ca cugc
Dap an C: boas: of something (v): khoe khoang ve cai gi


' y ''

Dap an D: pride on something/ doing something (v): tir hao ve dieu g i / lam

-

Dap an: A - in

-

Dien giai: Ta c6: in: trong, 6' trong + tliang, nam.

dieu gi.

Cac dap an con lai khong dung \k each dung giai t u tru'oc cac cum tir chi thai gian.
Dap an B: at: a, tai, vao luc + diem thoi gian

D E SO 4

'w<

M a r k the letter A , B, C or D on your answer sheet to indicate the correct
answer to each of the following questions.

Trich dan: at 7 o'clock
Dap an C : on: tren, 6' tren + ngiiy trong tuan.

Question 1: "Whose hat is it?" - "It's my


Trich dan: on Monday
Dap an D: for: trong (khoang thai gian hanh dong hoac sir viec xay ra).

A. is leaving

Trich dan: for 2 months ( t r o n g khoang 2 thang)

Question 3: He

C a u 47:
lis-

<

Ta CO cum tir: arrest somebody for somertliing/ doing something.
Cac dap an con lai khong dung ve each dung giqi tir trong cum tir nay.
Dap an: A - titled
Dien giai: title
"

(n) tira de (cua mpt quyen sach)
(v) mang tira de

B. was leaving

C. has left

B. might have

jjii^^j,., t

, /i,

D. left

C. could have

D . was going to

C. hasn't he

D. has he

Question 4: He owes you a lot o f money,
A. isn't it

B. doesn't he

Question 5: I am not ready,
B. and neither is she

C. and so is she

D. she is too

Question 6: He teU phoned while 1
A. am having

dinner.

B. had


C. have had

Cac dap an con lai khong hop ve nghTa c&n dien trong cho trong.

estion 7: " W o u l i you mind turning the fan on?" - "

Dap an B: call

A. Yes, please

( v ) : ggi, moi gpi, dugc goi la

D. sisters'

so he must be at the office now.

A. and she isn't too

C a u 48:
-

C. sister's

book the tickets, but he hadn't enough time to call at the cinema

A. would have

Dap an: D - for
Dien giai:


-

B. sisters

Question 2: it's an hour since he

thu'ong du-gc dung trong thi H T H T .

-

A- sister

"

B. No, thank you

C. Not at all

D. was having
"
D . N o , I ' m not
111


Question 8: We just whispered
A . so

B . in order that


Question 9: The clock
A . pointed

D . therefore

C. tells

D-show

Question 26:

A . chooses

B. pauses

C. rises

D. horse

Question 27:

A . decided

B. hatred

C. sacred

D . wanned

C. Having


D . Feeling

Question 28:

A . head

B. break

C. bread

D . breath

8.30.
B . says

Question 10:

C. otherwise

M a r k the letter A , B , C or D on your answer sheet to indicate the wOfd
^hose underlined part is pronounced differently from that of the rest j , , c-icli
of the following questions.

he heard the story.

tired, I took a short rest.

A. To fell


B . Being feeling

Question 11: The man
A . succeeded

to open the window at the back o f the house.
B . managed

C. forced

Question 12: The little girl wouldn't go to the sea
A . except

B . but

Question 13: Is anyone
A . let

''Wift

Question 15:

A . blood

B. tool

C. moon

D. spool


A . height

B. tme

C. tjdy

D. cliff

C. admitted

D . passed

M a r k the letter A , B , C or D on your answer sheet to show the underlined
part that needs correction.
Question 31: We entered into the room and turned the light on.
A

a more serious attitude toward your work.

B . grow

C. become

D . grow up

B . Lesser

C. Fewer

Question 16: I prefer being on my own

A . than to be

D . Few

C. to being

D . or be

A

B . Nevertheless

C. However

Question 18: John borrowed 14,000 dollars
A. o f

B . by

A . to cause
C.forcausing

D . from

B. o f having causes
;

D . that we cause

Question 20: What vegetable would you like,


C


4

D

B C

v:r..

D

^-

A

B

C

y ^ ' M

D

Question 35: Hardly had I gone out than it rained heavily.
A

passengers any inconveniences."


D

Question 34: N o sooner I had turned the T V on than she went out.

the bank.
C. at

Question 19: " W e apologize

D . Although

B

A

I have never been there myself, I have heard of it.

A . But

C

Question 33: Have you found a better hotel, haven't you?

with a big crowd o f people.

B . rather than be

B


Question 32: A lots o f people have left for the country to seek for job.

people go to football matches now than twenty years ago.

A . Less

Question 17:

D. i f

to fish in this river?

Question 14: Y o u w i l l have to

Question 29:
Question 30:

her father didn't.

C. also

B. allowed

A . develop

D . discovered

B

C


, ' " ''

D

M a r k the letter A , B , C or D on your answer sheet to indicate the sentence
that is closest in meaning to each of the following questions.
Question 36: I f you don't arrive on time, you won't meet him.

jj' iiv '

A. Unless don't arrive on time, you w i l l meet him.

?

.A

B. Y o u won't meet him provided you don't arrive on time.

A . peaches or carrots

B. peas or potatoes

C. Unless you arrive on time, you won't meet him.

= jdw *

C. tomatoes or pears

D. beans or apple


D. A r r i v i n g in time, you won't meet him.

Jftw .(:•

Question 21: He drew all his money
A. o f

the bank before he left.

B.off

C. out o f

Question 22: As the car is small, it's much more
A . expensive

B. poor

Question 23. I f only he

D.to

on petrol.

C. economical

'-"h 0

told us the truth in the first place, things woul

C. would have

D . should have

o f exchange at a bank.

B. value

Question 25: I've got time for a very quick
A. snack

C. 1 told you not to buy this book. It is worthless reading.
D. As this book's worthless reading, you don't buy it.

' "' ' '

Question 38: I can't answer your question because I haven't read that book
B . has

Question 24: Y o u ' l l get better
A . rate

'

B. I f I were you, I wouldn't buy this book. It is worthless reading.
D . economic

have gone wrong.
A . had


Question 37: Y o u shouldn't buy this book. It is worthless reading.
A. This book is worthless reading so, you don't buy it.

B. barbecue

C. worth

A. Not reading that books before, I can't answer your question.
D . charge

before I go.
C. feast

•before.
B. Not until I read that book before, I could have answered your question.
C. Had I read that book before, I could have answered your question.

D . picnic

' '•
'

D. Having I read that book before, I could have answered your question.
113


Question 3 9 : The computer helped me work more effectively.
A . According to the computer, I worked more effectively.
B. Thank to the computer, 1 work more effectively.


Question 45: Which o f the following statements is true according to the passage?
A. In many areas o f the world, architecture is the most important art.
B. Buildings o f various sorts affect us in different ways.

C. Because o f the computer, I worked more effectively.

C. Churches are the most attractive buildings in the city.

D . Without the computer, I wouldn't have worked more effectively like this.

D. Many people like to look at buildings, others do not.

Question 40: It won't make any difference if he doesn't come. We'll still leavt
Read the following passage and mark the letter A , B , C , or D on your

for D a Nang City.

answer sheet to indicate the correct word for each of the blanks from 46 to 50.

A . i f he doesn't come, we won't leave for Da Nang City.
f

Many people think o f computers as very modern . . . ( 4 6 ) . . . , products o f our
new technological age. But actually the idea for a computer was worked out over
, ^ 0 centuries ago by a man .. .(47)... Charles Babbage.

Read the following passage and mark the letter A , B , C , or D on your answer |
sheet to indicate the correct answer to each of the questions from 41 to 45.

drew up plans for several calculating machines which he called "engines". But


B. We w i l l leave for Da Nang City although he doesn't come.

'*'

C. We w i l l leave for Da Nang City so long as he comes.

sj,

D. Provided he comes, we w i l l leave for Da Nang City.

rf,

Whether we ealize it not, the arts are part o f the daily living o f everyone. The
arts are important in much that we see, hear and do. When we walk down a city
street or ride down a country road we pass buildings o f various sorts - houses,
stores, barns, churches. Some o f them look attractive and inviting, others do not.
However, in looking at building and reacting to them we are being concerned with
architecture, one o f the oldest and most important o f the many areas o f art.
Question 41: The author's purpose when he writes the above passage is:
A. to describe the beauty o f nature in the country.

Babbage was born in 1791 and grew up to be a brillilant mathematician. He
...(48)... the fact that he stared building some o f these, he never finished any o f
them. Over the years people have argued whether his machines would ever work.
Recently, however, the Science Museum in London has finished buiding an engine
...(49)... on one o f Babbage's designs. It has taken six years to complete and more
than four thousand parts have been specially made. Whether it works or not, the
machine w i l l be on show at a special exhibition in the Science Museum to
...(50)... people o f Babbage's work.

Question 46:

A . inventions

B. to describe the beauty o f the modern buidings in the city.

Question 47:

A . renamed

C. to point out the presence o f art in our every day life.

Question 48:

A . in spite

Question 49:

A . based

Question 50:

A . remember

D. to insist upon the important role o f architects.
Question 42: By "Whether we realize it or not" the author means:

=

B. advance


foi;
C. invent

D. inventor

B. known

C. calling

D. called

B. despite

C. event

D. though f

B. focus

C. depend

D. step

B. arrested

C. remind

D.accused


BO'.

A. whether we produce works o f arts or not.
HlTONG DAN GIAI C H I T I E T

B. whether we are artists or not.
C. whether we are conscious o f the fact or not.

^|ii,u 1: Dap an C - sister's

D. whether we accept the existence o f the arts or not.
Question 43: When a building looks "inviting", it
A. fascinates us more or less
B. asks us to attend a party held in it

C. it attracts us when we walk down a city street.
D. it effects us every day.
114

Ve mat ngir phap: Ta quan sat thay cau d l six dung cau hoi hk dau bSng
cua ai". Cau tra loi phai de cap den cai gi thuoc quyen so hiru cua

B. V i ca hai phuong an nay khong de cap den van dk s& him. Ta thay ca phuang
C va D deu

cap den so huu nhung phuong an D khong chon. V i sisters' la

'^'nh thuc so huu so nhieu. Trong cau de chi c6 mot cai mu nen sister's, hlnh thurc

important o f the many areas o f art because

B. thanks to it we can have attractive buildings.

+

:

toi."

ai hoac nguoi nao so huu cai g i . Dua vao d i l u nay, ta khong chon hai phuong an A

D. demands us to stay in it

A . without it we cannot have buildings o f various sorts.

Ve mat ngir nghia: ''Chiec wu ciia ai the? - Ciia chigdi

^hose

C. is ready to give a party
Question 44: According to the author, architecture is one o f the oldest and most

+

so ft la phu hop nhat.
^^u 2: Dap an D - left
.

Ve mat ngir nghTa: "Da mot tieng dong ho troi qua tir luc anh ay di khoi nen

S*o nay ch5c chSn anh ky da c6 mat tai van phong."

Ve mat ngu' phap: Quan sat cau de cho ta thay c6 since he

^
115


Giii nhanh CDBT TN Anh Van - NgO VSn Minh

Nlur ta biet, theo sau since c6 the la moc thai gian hoac mot m^nh de c6 dpng tu
chinh chia thi qua khiV don. Cac phirong an con lai khong phu hop.
A. is leaving: thi hien tai tiep dien
B. was leaving: thi qua khu tiep dien

+ Ve mat ngu phap: Quan sat cau d^ ta thay He phoned while
Qiid khudan

japg

+ while + qua khi'ttiep dien: dien ta hanh dpng xay ra truWc va keo

dai so v o i hanh dpng khac xay ra sau va cham dut trong qua khu.

C a u 3: Dap an D - was going to

Vay D la lira chpn duy nhat.

Vc mat ngCr nghTa: ''Anh dy c1ci chr clinh inua vai (dm ve nay nhimg anh uy

Cac dap an con lai khong phu hop v i :


khong CO thoi gian etc goi den rap phiin.'"
+

Dfjy

cua cau true:

''"

C. has left: thi hien tai hoan thanh
+

CSu 6: Dap an D - was having
+ Ve mat ngu nghTa: ''Anh dy goi di^n khi toi dang an toi"

A. am having: thi hi^n tai tiep dien

Ve mat ngir phap: Ta quan sat thay ngay sau cho trong can dien la mpt dong

tir nguyen mau (book). Dira vao cac phirong an dap an da cho, ta nhanh chong ciion

B. had: thi qua k h u don

dugc dap an D : was going to.

C. have had: thi hien tai hoan thanh

do

Be + going to + V nguyc'n ven - dien ta dir djnh co kha nang xay ra

Cac dap an con lai khong phij hop v i khong ton tai dang thuc nay, ma phai la
would/couid/might + have + V3/ed.

; j | yj,
. ris v •
=

^xmt

f^, •
,

Lu'u y : While - trong k h i , khong bao g i o diing kem v o i thi qua khu don va thi
hien tai hoan thanh. N o chi dupe dimg mpt trong hai thi sau: hien tai tiep dien va
qua khir tiep dien.

;s.^;

Cau 7: Dap an C - N o t at all

fi».(.3'i H ,:'

Cau 4: Dap an B - doesn't he

Xet nghTa cac pluro'ng an dap an:

.f u

, . « ;j


+

Ve mfit ngif nghTa: ""Anh dy na ban nhieii tien phdi khong?"'

A. Yes, please: Vang, lam on.

+

Ve mat n g u phap: Quan sat cau van da cho ta thay c6 dang cau hoi duoi.

B. No, thank y o u : Khong, cam o n .

/ .,

Trong cau hoi duoi, neu menh de chinh a the khang djnh thi phan hoi duoi a the

C. Not at all: Khong co g i , dupe dimg nhu mot I6i dap cho I6i de nghj Ijch sir

phu djnh va ngugc lai. Ta xet thay menh de chinh ciia cau de c6 dong tir chinh chia

(Would you mind...?) hoac loi cam on (Thank you).

Of the kh5ng djnh (owes) nen phan hoi duoi ta phai diing hinh thuc phu djnh cua tro
dpng tir. NghTa la ta loai nhanh chong phuong an D : has he.
Ta lai thay cau van sir dung dpng tir thuong chia thi hien tai don (owes). N h u ta

§

D. No, I ' m not: Khong, toi khong


^,

biet, neu menh de chinh sir d^mg dpng tir thuong duy nhat thi phan hoi duoi phai

Dira vao ngir nghTa ciia cau van, C la lira chon duy nhat diing.

mugn t r g dpng t u tuong ung voi thi va chu ngu cua cau do. Trong truong hgp nay

+

ta phai mirpn trg dpng tir does va them not sau no, vay B la dap an duy nhat dung.
C a c dap an con lai khong phu hop vi:
A . isn't it: day la dang hoi duoi d the phu djnh cua dpng t u tobe (ap dyng cho
m^nh de chinh sir d^ing dpng t u tobe the khang djnh).
B. hasn't he: day la dang hoi duoi d the phu dinh cua dpng have/has.
Cau 5: Dap an B - and neither is she.
+ V e mat ngu nghTa: 'To/ chua sdn sang vd co dy cung vay.'"
'^'^ + Ve mat ngu phap: Ta nhan thay rang cau hoi nay thupc dang dap lai cung
khong dung voi Neither. M p t dieu toi quan trpng can phai nha do la Neither khong
bao g i a dung voi hinh thuc phu dinh v i ban than t u nay da mang nghTa phu djiil''
Ro rang la cau de dang sir dyng thi hi^n tai don, the phu djnh ciia dpng t u tobt-'
Di^ra vao ngii)en tac da neu tren, phuong an A : and she isn't too, sai hinh thuc v'
khi dimg too de dap lai cung vay thi ta phai luon dung hinh thuc khang djnh. T'''
cung loai C: and so is she va D : she is too v i "so va t o o " chi dupe dung trong ca^'
kiifing dinh. Vay B la dap an duy nhat dung.

Dich nghTa: "(\'ini phien ban hat cdi may c/uat dirge khong? - Khong co giT

Cau 8: Dap an C - otherw ise


,
7

Xet cac dap an:

:,

"

A. so ( + clause): v i vay
B. in order that (+ clause): de
C. otherwise (+clause): neu khong thi
D. therefore ( + clause): do do

^.
j t
'



Dya vao ngfr nghTa cau van da cho, chi co C la dap an duy nhat diing.
+

Dich nghTa: "Chi'ing la nen noi nho thoi nen khong anh dy iighe dirge edu

chtiyen."
Cau 9: Dap an C - t e l l s
+

^


Ai«\Vu»A,v«.,

Ve mat ngu nghTa: "Kim dong ho diem 8 fflVr 30phiil."

'

Trong each diing thong thuong, khi noi dong ho diem may gio ta phai diing chir
" t e l l " . Vay C la lira chpn duy nhat.
Cac dap an con lai khong dung vc each dung tir.
' ^
A. pointed: chi
B. says: noi
D. show: the hien

' yniifo)
•;
,^

„; ^

• t(i(lr\ '•^
::ifm^l^0i,i^.:!ii^ ai^117
:
i>,(*^;iq|!lJ-f-^


+

Cau 10: Dap an D - Feeling

+

Ve mat ngCr nghTa: "To/ c1a nglii n^^ai mot chut do cdiii thdy

Ve mat ngCr nghTa: "Ngay nay so lugng khan gja di xem cac tran dau bong

it hon hai muai nam truac."

met"

+ Ve mat ngir phap: Ta quaii sat tliay sau vj t r i can dien ia tinh tir tired (met
moi). Nhir ta biet, tinh tir tired chi di vai dong tir tobe hoac dpng tir "feel". Do vay
ta loai nhanh chong dap an C: Having.
Ta dung V - i n g + adj dau cau de bieu dat y nghTa cho menh de chi ly do. Vay D

+

Ve mat ngCr phap: Quan sat cau van da cho ta thay c6

t i l l hi?n

than

Cau van

thurc so sanh hon. Ta loai ngay dap an D . Few (khong c6 hinh thirc so

sanh) va C. Lesser (khong ton tai hinh thuc nay). Hon nua, people trong cau van la
danh tir dem dupe so nhieu. Ta loai tiep tuc dap an A . Less v i no la dang so sanh it


la dap an dung duy nhat.

jj^n cua little (little + danh tir khong d^m dupe). Vay C. Fewer (few + danh t i j dem

C a u 11: Dap an B - managed

^irpc s6 nhieu) la dap an duy nhat dung.

,.4 ,}\

Cau 16: Dap an C - to being

+

Ve mat ngiT nghTa: ''Ong dy dd xoay sd de ma cdnh ciai so sau iihd."

+

Ve mat ngu phap: Quan ctRi van da cho thay c6 The man ... to open. V i tri

+

Ve mat ngiT nghTa: "T6i thich a mot minh han Id tu (dp nai dong nguai.'"

can dien la mot dong tir thuoc dang V + Vto inf. Ta loai nhanh chong cac dap an:

+

V§ mat ngif phap: Cau de c6 dang cua cau true:


A . succeeded v i ta c6: succeed + in + V-ing/S.th: thanh cong trong viec gi; C .
forced vi ta c6: force S.o to do S.th: bupc ai do lam g i ; D. discovered v i ta c6
discover S.th: phat hien ra cai g i , khong phCi hop nghTa.
Vay B (manage + V to-inf: xoay x o de lam gi) la dap an duy nhat dung.

S + prefer + V - i n g + to + V-ing: thich lam viec nay hon lam vi?c kia
Vay C la lira chpn duy nhat.

^, ^

Cac dap an con lai khong phii hop ngu phap cua cau.

j
r

Cau 17: Dap an D - Although

C a u 12: Dap an D - i f
+

Xet cac phuong an dap an:

Ve mat ngiT ^^hTa: "Co he se khong di bien neu bd c6 ay khong di."

A. But: nhung, tuy nhien - "but" dung ik noi hai menh de doc lap dien ta y

Xet nghTa cac dap an:
A . except: ngoai trij'

B . but: nhung


C. also: cung vay

D. if: neu

nghTa trai nguoc nhau. " B u t " thuong theo sau dau phay va no bat dau cho menh de
thu hai.
B. Neverthless: tuy nhien - "nevertheless" dung de dien ta hai menh de mang

Dira vao nghTa cua cau van da cho va nghTa cua cac phuong an dap an, ta chon
D la dap an dung.

nghTa trai ngupc nhau, thuong dung dau cau va sau dau phay hoac dirng cuoi cau.
C. However: tuy nhien, nhung - "however" dung dd chi sy tuong phan, c6 the

C a u 13: Dap an B - allowed

dung dau cau, giua cau hoac cuoi cau. K h i dirng d5u cau thi ngay sau However ia

+

Ve mat ngu nghTa: ''Moi nguvi dmrc phep cau cd & .song nay khong?''''

+

Ve mat ngfr phap: Quan sat cau vSn da cho ta thay c6 ... to

fish

adj hoac adv.

Ta da

biet allow S.o + V t o inf: cho phep ai la g i . Ta loai ngay phuang an con lai v i :
A . let S.o + Vnguyen ven
C. admit + V - i n g : thira nhan la gi
D. pass (v) vuot qua/bang qua, khong hop nghTa
C a u 14: Dap an B - allowed
+

.

Ve mat ngi'r nghTa: ''Ban se phdi c6 thai dg tich cue hem nita doi vai cong

D. Although: mac dii - "although" diing d6 chi sir tuang phan, thuong dirng dau
cau hoac dirng giua cau lam lien tir n6i. Truoc va sau "although" khong co dau
chamcau.
Dua vao nhung lap luan tren, ta chon D la dap an duy nhat dung.
+

Dich nghTa: "Mac dii toi clura bao gio dat chan den do nhung toi da tirng

nghe ke vk no."
Cau 18: Dap an D - from

viec cua ban than.''''

+

V l mat ngu nghTa: "John dd vay ngdn hang 14 ngdn do.'"


Xet nghTa cua cac phirong an dap an:

+

Ve mat ngu phap: Ta c6 cum: borrow S.th from S.o/S.th: muan cai gi tir

A . develop: phat trien (mot vdn de nao do Ion hon)
B. grow: phat trien (ve s6 lu-gng va chat lirong)
C . become: tro iien
D. grow up: phat trien/tru-ong thanh (dung cho nguoi va dpng vat)
Can ci'r vao nhung phan tich a tren, A la dap an duy nhat dung.
C a u 15: Dap an C - Fewer
118

ai/cai gi
p.,,

,

,

^,

^

?j|)ttm a«M

^.

U la dap an duy nhat dung.


, . ,

Cac dap an con lai khong phu hop.
CSu 19: Dap an C - for causing

tf':'-^':i-,:ti

,.M-q»<-.'af,;/f

^

,

>- i

+

Ve mat ngu nghTa: "Chung toi xin loi vi gay cho hdnh khdch nhieu hat tien."

+

Ve mat ngu phap: Ta c6 cum: apologize + for + V-ing: x i n loi vi lam vice gi
119


Cac dap an con lai khong phu hop vi:

C ici dap an diiy iihat dung.


B. value (n): gia trj

Cac dap an con lai khong phii hop.
C i i u 20: Dap an B - peas or potatoes
+

C. worth (a): dang gia

ftiwO ;

V6 nu'it ngu- ngiiTa: "Ban l/iicli loai ran qua nao, dgu Ha Lan hay khoai tdy?"

A. peaches or carrots: qua dao hay ca rot (hai lo^i qua nay khong dong dang,
dao la mot loai trai cay con ca rot xep vao danh muc cac loai rau)
B. peas or potatoes: dau Ha Lan hay khoai tay (hai loai qua nay deu duoc xep
C. tomatoes or pears: ca chua hay qua le (hai loai qua nay khong dong dang, ca
D. beans or apple: dau nanh hay tao (hai loai qua nay khong dong dang, dau

A. snack: bira an nhanh, bua an qua loa

B. barbuecue: v i nuong

C. feast: bira tiec

D. picnic: chuyen da ngoai

Jlnhanh" nen ta chon A la dap an dung duy nhat.
Ve mat ngir nghTa: 'To/ chi an qua loa Irirac khi di."

Cau 26: Dap an D - horse

V i ta c6: /se/ trong horse dirge phat am la /s/; /es/ trong cac tir: chooses, pauses,

nanh xep vao danh muc cac loai rau con tao la mot loai trai cay)
Quan sat can de ta thay c6 "vegetables: rau (noi chung)". TiJ nhung phan tich

rises dugc phat am la /iz/.
Cau 27: Dap an A - decided

tren, ta chon B la dap an duy nhat dung.
Ciiu 21: Dap an C - out of

V i ta c6: /ed/ trong decided dugc phat am la /id/; /ed/ trong cac tir con lai:

Ve mat ngfr ngliTa: ""Anh la da n'lt liel lien gm a ngm

hang trm'rc khi anJi ta

decided, hatred, sacred dugc phat am la /d/.
Cau 28: Dap an B - break

ra cHr
Ta xet nghTa ciia cac gioi tir cho ben duoi khi di vai draw:

V i ta c6: /ea/ trong break dugc phat am la /ei/. /ea/ trong cac tin con lai: head,

draw to S.th: hap dan ai hoac lam ai quan tam

bread, breath dirge phat am la /e/.

-


draw out o f S.th: rut (tien) t i r . . .

Cau 29: Dap an A - blood

-

draw o f f S.th: lay cai gi tir mot dir trfr Ion hon
;,

C a u 22: Dap an C - economical
B. poor (a): ngheo

C. economical (a): tiet kiem

D. economic (a): thuoc ve kinh te

Dira vao ngliTa cua cau van da cho, ta chon C la dap an dung.
Dich nghTa: "Boi vi kich ca chiec xe nho nen no liel kiem nhieu nhien lieu hanT

C a u 23: Dap an A - had
Ve mat ngu nghTa: " G w nia anh dy noi cho chung toi nghe sir Ihcit ngay h'lc

dau ihi inoi ihi'r c6 Ihe dd khong di sai hirovg.''
Ve mat ngfr phap: Ta quan sat thay cau d6 c6 If only

gia ma

Nhir ta


biet I f only + clause ( Q K H T ) , clause (S + could/would + have + V3/ed) de dien ta
mot dieu iroc muon khong c6 thuc trong qua khir.
Cac dap an con lai khong phu hop vh nghTa va cau true ngir phap.
C a u 24: Dap an A - rate
Ta CO cum: rate of exchange: ty gia hoi doai
+

moon, spool diroc phat am la /u:/
V i ta c6: / i / trong cliff dugc phat am la / i / ; /if, /ei/ trong cac tir con lai: height,

A. expensive (a): dat

+

w

Cau 30: Dap an D - c l i f f

Xet ngliTa cac dap an:

+

m>f.

V i ta c6: /oo/ trong blood dugc phat am la /A/; /OO/ trong cac tir con lai: tool,

Va) C la lira chon duy nhat.

120


Xet nghTa ciia cac dap an:

^ +

chua xep vao danh miic cac loai rau con le la mot loai trai cay)

+

,
^/

Dira vao ngO nghTa cua cac phuong an va cau van de cho c6 "a very quick...: rSt

vao danh miic cac loai rau)

-

,

D. charge (n): phi, tien phai tra
Cau 25: Dap an A - snack

Xet cac dap an:

+

. f,,,

Dich nt>hTa: "Ban se hm'mg muc ty gia hoi dodi a ngdn hang cao hanT


fine, tidy dugc phat am la /ai/.
Cau 31: Dap an B - into

.n

X

+

Ve mat ngir nghTa: '"Chung loi birac vaopheng vd bdt den len."

+

Ve mat ngir phap: Ta khong dinig gioi tir theo sau enter.

• ni; ;
ij'. '

Ta c6: enter = come in = go in: vao, den


C a u 3 2 : Dapan A - A l o t s o f - > A l o t o f
+

Ve mat ngu nghTa: "'Nhieu rigum rai khoi viing que di ttm vice Idm."

+

V l mat ngir phap: A lot o f + danh tir dem dugc so nhieu


Cau 33: Dap an A - Have you - > You have

'

ivi:;/;
''

\&-fhm


.

+

Ve mat ngiT nghTa:

+

Ve mat ngir phap: Cau van da cho c6 dang cau hoi duoi. N h u ta hik, trong

'Ban dd tim dirac mot khdch sgn tot hon roi phdi

'^U hoi duoi hinh thuc nghi viin khong dirge dimg d menh de chi'nh, chi dung cho
121


CtyJwmgrnrPWH Khang Vlgt

phan hoi duoi. Do vay, ta phai sira lai hinh thuc nghi van (Have you?) trong m^nh

dS chi'nh sang hinh thuc khang djnh (You have).
Cau 34: Dap an B - I had
had I
+ Ve mat ngir nghTa: "To/ vita mai bat ti vi thi cd dy cli ra ngoai.''+
IV

Ve mat ngir phap: Ta c6 cau true:
No sooner + had + S + V3/ed + than +

: VCra mai

thi

Cau 35: Dap an B - than —» when
+ Ve mat ngir nghTa: '"Anh dy c1i khoi khong bao Idu thi trai mua to!"
Ve mat ngfr phap: Ta c6 cau true:
Hardly + had + S + V3/ed + when + QKD: khong bao lau thi
= S + had + hardly + V3/ed + when + QKD
Cau 36: Dap an C - Unless you arrive on time, you won't meet him.
+ Cau goc mang nghTa: ''Neu ban khong ilen clung gia, ban se khong ggp dieac
anh dy."
+ NghTa cau dap an: ''Neu bgn khong den dung gia, bgn se khong ggp duac
anh ay:'
A:.
• + Ve mat ngij phap: Cau goc de cho sur dung cau dieu kien loai I : If + S + hieii
tai don (phu djnh), S + will + not + V nguyen ven. Dua vao cac phuang an dap an
cho ben duoi, chi c6 C la dap an dung duy nhat vi ta c6:
if

not


= Unless: Neu khong

thi

Cac dap an con lai khong dung vi:
A. Unless you don't arrive on time, you will meet him: sai cau true cau, ta
khong dung the phu djnh trong m^nh de di vai Unless vi ban than Unless da mang
nghTa phu djnh.
B. You won't meet him provided you don't arrive on time: Ban se khong gap
dupe anh ay mien la ban khong den dung gia - sai nghTa cau goc ban dau.
Luu y: Trong cau dieu ki^n loai I , ta dung "Provided/ Providing (that)" d l thay
the cho I f va mang nghTa la "mien sao/ mien la". Dong tir trong m^nh de
"Provided/ Providing (that)" van dugc chia binh thuang.
D. Arriving in time, you won't meet him: Den dung gia thi ban se khong gap
dugc anh ay - ngugc nghTa cau goc ban dau.
Cau 37: Dap an B - I f I were you, I wouldn't buy this book. It is worthless readingV + Cau goc mang nghTa: "Bgn khong nen mua cuon sdch nay. DQC no chdng cd
ich kfi gi."
+ NghTa cau dap an: "Neu toi la bgn, toi se khong mua cuon sdch nay. DQC no
chdng CO ich lai g/."
122

Should + Vnguyen v ? n : nen lam gi
= If I were

= S + had + no sooner + V3/ed + than +

+

+ Ve mat ngir phap: Ta quan sat thay cau goc de cho c6 shouldn't

khong
fi^n.-- Ban chat cua no l a m p t lai khuyen. Trong truang hgp nay, ta c6 cac cau true
tu-crng dong nghTa:
: neu toi la

= Had better + Vnguyen ven: tot han nen


|

;^^,|.

Dya vao ngir phap va ngir nghTa, B la lya chgn duy nhk.
Cac dap an con lai khong diing vi:

1 ,'
'

A. This book is worthless reading so, you don't buy it: Doc cuon sach nay
ch5ng C O ich Igi gi ca vi vay ban dirng mua no - cau van khong the h i ? n dugc lai
khuyen (don't...)
C. I told you not to buy this book. It is worthless r e a d i n g : Toi da bao ban khong
mua cuon sach nay. Dgc no chang c6 ich Igi gi ca - cau van khong the hi?n nghTa
cua mot lai khuyen ma la mgt lai trach cho chuyen da roi.
f^*:, ,
D. As this book's w o r t h l e s s r e a d i n g , you don't buy it: vi cuon sach nay dang
dugc dgc. ban khong dugc mua no. Khong dung y nghTa cua cau goc.
Cau 38: Dap an C - Had I read that book before, I could have answered your
question.
)

+ Cau goc mang nghTa: "Toi khong the trd lai cdu hoi cua bgn vi toi chira dgc
quyen sdch do truac ddy.
+ NghTa cau dap an: "Neu toi doc quyen sdch do truac ddy thi toi dd cd the trd
lai cdu hoi ciia bgn. "
Ta xct Ian lugl cac phuang an dap an:

' '

A. Not r e a d i n g that books before, I can't a n s w e r your question: Khong dgc
nhimg cuon sach nay truac day nen toi khong the tra lai cau hoi cua ban - diing
nghTa so vai cau goc ban dau nhung sai ngir phap vi cau goc chi su dung "that
book: quyen sach do", cau viet lai su dung "that books: nhirng quyen sach do".
B. Not until I read that book before, I could have answered your a n s w e r : sai ve mat
figu' phap vi khi c6 Not until dirng dau cau thi m^nh con lai cua cau phai dao ngir. .
C. Had I read that book before, I could have answered your question: Neu toi
quykn sach do truac day thi toi da c6 t h i tra lai cau hoi cua ban - cau van c6
cua cau true:
Had + S + V3/ed, S + would/could + have + V3/ed: Neu
= I f + S + had + V3/ed, S + would/could + have + V3/ed

thi

D. Having I read that book, I could have answered your question: khong ton t^i
^^ngcau nay.
Tir nhung lap luan tren, ta chgn C l a dap an duy nhat dung.
^Su 39: Dap an D - Without the computer, I w o u l d n ' t have
E f f e c t i v e l y like this.

worked


more


+ Cau goc mang ngliTa: ''May vi tinh da gitip toi lam vi^c hieu qua han.'"
+ NghTa cau dap an: ''Khong c6 may vi tinh, toi da khong thi lam vi^c M^u (jun
nhuvgy."
Xet cac dap an:
A. According to tiie computer, I worked more effectively: Theo cai may vi tinli
nay, toi da la viec hieu qua hon - sai vi ta chi dung According to S.o/ S.th: theo ai
do (the iii^n y kien ca nhan)/ theo nguon tai lieu nao do.
B. Thank to the computer, 1 worked more effectively: Nha vao may v i tinh, toi
da la viec hieu qua han - dung ve nghTa nhung sai ve ngu phap vi ta c6 dang dung
la Thanks to + noun: nha vao cai gi.
C. Because of the computer, I worked more effectively: Bai vi may vi tinh nen
toi da la viec hieu qua han - sai ve nghTa.
?> n •
D. Without the computer, I wouldn't have worked more effectively like this:
Khong CO may vi tinh, toi da khong thk lam viec hieu qua nhir vay. Cau nay diiiig
\k mat ngCr nghTa Ian ngu phap: Without + N/N.P: Nha vao cai gi/dieu gi, duo
dung de bieu dat y nghTa phu djnh cua m?nh dk gia sir trong cau dieu kien loai III.
Vay D la lira chpn duy nhat.

Cty TNHH MTV DWH Khang Vi§t

lihi clu'ing ta tan bo tren duang pho hoac d?p xe ve cac mien que, bang qua cac toa
,iha, cac ciia hang. chu6ng nuoi gia sue va nha tha. Nhieu nai trong so nhung dja
di^ii ke tren c6 sue loi cuon va hap din, con nhiJng nai khac thi khong. Mac dhu
vay. khi nhin vao mpt toa nha thi chiTng ta nghT ngay den mpt trong nhung ITnh vyc
quan trong nhat va lau dai nhat, do la kien true.
lUjiii^^..

,
Cau 41: Dap an C - to point out the presence of art in our everyday life.
+ Dich nghTa cau hoi va dap an:
Muc dich cua tac gia khi viet bai lu^n tren la:
A. Mieu ta ve dep thien nhien cua lang que.
B. Mieu la nliu'ng toa nha hien dai a thanh pho.

' i*-^^

C. Chi ra sir hien hUu ciia cac tac pham nghe thuat trong dai song thuong ngay.
D. Khang djnh tarn quan trong cua cac nha kien true su.
C la lua chpn duy nhat (dong 1).

^'^''^'^^

Cau 42: Dap an C - whether we are conscious of the fact or not.
+

Djch nghTa cau hoi va dap an:

Cum "Whether we realize it or not" nghTa la:

Cau 40: Dap an B - We will leave for Da Nang City although he doesn't come

A. Lieu con nguai c6 t^o ra cac tac phhm ngh? thuat hay la khong.

ig

+ Cau goc mang nghTa: ''SH khong c6 gi thay ddi niu anh ay khong den. Chiing
ta se vdn den thanh pho Da Nang.'"

+ NghTa cau dap an: "Z)M anh dy khong den thi chmg Ic. ciing .se den thanhphd

B. Lieu con nguoi c6 phai la nhung hpa sT hay la khong.

,n ?!; ;

DaNSng.''
^' •
Xet cac dap an:
A. If he doesn't come, we won't leave for Da Nang City: Neu anh ay khong den
thi Chung ta se khong den thanh pho Da Nlng - sai nghTa so voi cau goc ban dau.
B. We will leave for Da Nang City although he doesn't come: Du anh ay khong
den thi chiing ta se den thanh pho Da NSng - cau van dung ve mat ngu nghTa i:in
ngu phap: Although + clause: mac du ...
C. We will leave for Da Nang City as long as he comes: Chung ta se rai khoi
thanh ph6 Da NSng miln la anh ay den - sai nghTa so vai cau goc ban dau.
D. Provided he comes, w e will leave for Da Nang City: Mien la anh ay den tin
chimg ta se den thanh ph6 Da Nang - sai nghTa so voi cau goc ban dau. Trong cau
dieu kien loai I , ta c6:
4i

Provided/ Providing (that): mien la = I f

Tir nhiJng lap luan tren, B la dap an duy nhat dung.
* Bai djch de n^hj:
. vu s
Cac tac pham ngh? thuat g5n bo khang khit vai dai song thuong nhat ciia chi'm?
ta nhung li?u con nguoi c6 nhan ra dieu do hay khong. Cac tac pham nghe tliua'
anh huong sau rpng den muc cluTng ta c6 the nhin, nghe va hanh dgng. Co the 1^


C. Li^u con nguai c6 nhan ra dugc thuc te hay la khong.
D. Li?u con nguai c6 chap nhan sy t6n tai cua cac tac phSm nghe thuat hay la khong.
C la dap an duy nhSt dung.
Cau 43: Dap an C - fascinates us more or less.
+

'I- «i

Dich nghTa cau hoi va dap an:

Khi mot toa nha nhin c6 tinh hap dan, no



A. loi cuon chung ta nhieu hay ft
B. doi hoi chung ta phai tham du ti^c to chuc t^i do
C. sin sang de to chuc ti|c

' mq&i)

D. yeu cau chung ta a do

'

A la lya chpn duy nhat.
Cau 44: Dap an D - it affects us every day.
+ Djch nghTa cau hoi va dap an:
Theo y tac gia, kien true la mot trong nhung ITnh vvrc nghe thuat quan trong nhat
Va lau dai nhat vi:
A. khong CO kien true thi chung ta khong th6 xay nhieu cong trinh khac nhau.

B. nha vao kien true nen chiing ta mai c6 dugc nhiJng cong trinh kien true dep.
C. kien triic thu hut chiing ta khi tan bp tren duong ph6.
., ^
D. kien true anh huang den chiing ta hang ngay.
pSu 45: Dap an B - Buildings of various sorts affect us in different ways.
I + Djch nghTa cau hoi va dap an:
• ,:. • .
.

125


DE SO 5

Cau nao duai day la dung theo doan van tren.
A . K i ^ n true la mpt ITnh vyc ngli? thuat quan trong a nhiSu nai tren the gioi.
B. Nhung cong trinh kien triic khac nhau anh huang den chung ta theo nhieu
each khac nhau.
C. Nha tha la cong trinh kien true c6 sue hut nhat tai cac thanh pho.
D. Nliieu nguoi thich nhin ngSm cac cong trinh kien true, con nhung nguoi khac
thi khong.
B la lira chon duy nhat dung.
*

B a i djch de nghj:
Nhieu ngu-oi cho rSng may vi tinh la mpt phat minh circ ky hien dai, la san

phkm tien phong cua the he cong nghe m o i . Tuy nhien tren thirc te, y tuong phat
minh ra may v i tinh do ong Charles Babbage thirc hien hai muai nam truoc.
Babbage sinh nam 1791 va la mot nha toan hoc 16i lac. Ong vach ra nhung kc

hoach khac nhau ve cac cong trinh nghien curu ve dong ca. Mac du ong la nguoi
khoi xuong nhung cong trinh nghien ciru tren nhung ong chua bao gia hoan thanh
mot trong s6 cac cong trinh do. Da nhieu nam troi qua nhung nguoi ta van con
tranh cai lieu nhung may moc do ong sang che c6 hoat dong hay chua. Tuy nhien,
thai gian g5n day, Vi?n bao tang khoa hoc a Luan Don da hoan thanh mot cong
trinh nghien cuu ve dong ca dua tren mpt trong nhung thiet ke cua Babbage. Da
phai mat 6 nam de hoan thanh va san xuat ra bon nghin san pham nhung lieu no c6
hoat dong duoc hay khong. Chiec dong ca se v i n dugc trung bay tai mot trien lam
dac biet nao do tai Vien bao tang khoa hoc de nhac nha cong chung nha ve tac
pham ngh? thuat ciia Babbage.

B. advance (n): su tien bg
D. inventors (n): nguofi phat minh

D. hour

Question 2:

A . music

B. present

C. reason

D. sight

Question 3:

A . translated


B. washed

C. laughed

D. watched

Question 4:

A . father

B. brother

C. mother

D. throw

Question 5:

A . jdea

B . skm

C. find

D. flight.

M a r k the letter A,B, C or D on your answer sheet to indicate the correct
answer to each of the following questions.
Question 6: M y father told me
A . don't make


noise so as not to bother her.

B . not to make

B. known: dugc biet den
D. called: dugc ggi la

C. don't do

D . not to do"

Question 7: M y children are the most important
A. with
A. delayed

C. must delay

A. are

D . delay

C. working

D . to working

searching for the man who stole the car.
B. has been

C. is


D . was

1 had finished my homework.

B. i f

C. as i f

Question 12: I am going

D. A and B are correct

tomorrow morning.

A. have my room painted

C. need painting my room
D . painted the room
interesting that I have seen it many times.

B. very

Question 14: American women are used
A. to be

D . about

six days a week.


B . work

Question 10: The police

C. to

the meeting until tomorrow.

B. w i l l delay

Question 9: We are accustomed
A. to work

me.

B. for

Question 8: It is important that we

A . such

Xet ngliTa cac phuong an dap an:

B. to being

C. too

D . so
independent.


C. to have been

D. being

Question 15: She eats so much that she is now suffering from

D la dap an duy nhat dung.

A. weigh

C a u 48: Dap an B - despite.
Ta CO cum: despite the fact that + clause: mac dij
C a u 49: Dap an A - based. Ta c6: based on S.th: dua tren cai gi
C a u 50: Dap an C - remind. Xet nghTa cac phuang an dap an:
A . remember: nha

B . arrested: duoi bSt

C. remind: nh5c nha

D. accused: buoc toi

C la dap an duy nhat dung.

C. sound

Question 13: The video film is

C a u 47: Dap an D - called


C. calling: dang ggi la

B . account

B. the room painted

B la dap an duy nhat dung.

A . renamed: dugc dat ten la

A . tour

A. whether

Xet ngliTa cac phuang an dap an:
C. invent (v): phat minh

Question 1:

Question 11: They asked me

C a u 46: Dap an B - inventions
A. inventions (n): phat minh

M a r k the letter A , B , C or D on your answer sheet to indicate the word
^hose underlined part is pronounced differently from that of the rest in each
of the following questions.

Question 16: "Please
children.

A . believe in^*^*'*'-*

B. weight

C. overweight

'
^

D. being weight

everything while I am not at home," she said to her
B. bring about

C. tear o f f

D. take care o f

Question 17: Every year many people in the countryside leave for Ho Chi Minh
City to find j o b . Some o f them are teenagers
A. Another
B. The others
C. Others

are adults.
D . The other
won 'vrmri ai siJ^s
127

126


1


PiQuestion 1 8 : He presented his topic
than we expected.
A. well
B. good
C. fluently
D. better
Question 19:
my friends nor 1 am thirsty.
A. Either
B. Both
C. Among
D. Neither;)^. :
Question 20: it
me two hours to repair the roof of the house.
A. took
B. spent
C. waste D. passed
Question 21:1
photographs.
A. enjoy taking
C. don't mind taking a
B. to take
"
D. hate taking one
Question 22: There
an increase in road - accidents in the last few years.

A. have been
C. are
B. will have been D. has been
Question 23: If I had enough money, 1
the informative book.
A. will buy
C. had bought
B. would buy
D. am buying
Question 24: I
that he go to see his doctor as soon as possible.
A. ordered
B. informed
C. regretted
D. suggested
Question 25: He doesn't like the picture and
A. so am I
B. neither did I
C. neither do I
D . I am, too
Question 26: - "How is the book?"
A. It's expensive
B. It's written in English
Question 27: - "Are there any seats left?"
A. Every table is taken
B. Nobody is here

Question 32: If the plan is related to the people,
A. they would have gone ahead with it
B. he had to solve the problem

>< A
C. they are very happy to know that
.wif:^iii>;j ,
,Ji
D. the government will go ahead with it
Question 33:
he had met me last night.
^
A. He told as i f
'"
C. They suggested that
B. He asked me i f
*
D. If he had been there
if|aa« . J
flbuestion 34: The women can hold large containers of water on their heads
^
A. as they walk
B. so as to be strong enough for
C. when they walked
D. while they were walking
uestion 35: The container of water is too
A. very heavy for us to carry
C. heavy for me to carry it
^^^^'^'
B. heavy for me to carry
D. heavy that I can't carry it

t


Mark the letter A, B, C or D on your answer sheet to show the underlined
part that needs correcting.
Question 36: Most of the water we use come from rivers, lakes, and the atmosphere.
A
B
C
D
Question 37: These doctor checked my blood pressure during a physical examination

C. It's good
D. They're old

A
B
C
D
Question 38: You can't blame with what happened because he wasn't here last night
'
A
B
C
D

C. The table needs painting
D. Yes. There are two by the door

Question 28: - "Can I give you a soft drink?"

Question 39: She always talks to her baby as though she is an adult.
A

B
CD
Question 40: I would rather you stay at home than went on a picnic.
A

A. Yes, please. Orange juice is fine
B. It's very kind of you
Question 29: - " I approve of your plan."
A. Oh, no

B. Thanks

C. Ok. You can
D. I feel quite fine

C. Yes, you did

D. It belongs to you

Question 30: - "He likes watching films on TV."
A. Neither do I

B. So am I

C. So do I

D. I am, too

Mark the letter A,B, C or D on your answer sheet to indicate the correct
answer to each of the following questions.

Question 31: Paula has been ill for two months,
A. she is working hard now
B. although she is taking the medicine
C. but she is better now
D. because she stayed up late last night

B

C

jj

D

Read the following passage and mark the letter A, B, C, or D on your answer
sheet to indicate the correct answer to each of the questions from 41 to 45.
Smoking causes lung cancer, which is the number one cancer among men.
Ninety percent of the people who get lung cancer die. Smoking is also the leading
cause of mouth cancer, tongue cancer, and throat cancer.
Many smokers have heart disease and pneumonia. Smoking causes about one
fnillion early deaths in the world every year.
Smokers not only harm themselves but also harm others. Smokers breathe
sinoke out into the air.
They breathe it out on their children and on their wives or husbands. Children
^hose parents smoke have more breathing and lung problems than other children,
'^omen who are married to smokers are more likely to have lung cancer than those
|*iarried to non - smokers.

.
129



We are all aware tliat smoking is bad. So why do people smoke?
Question 41: Every year, smoking causes about one million
A. killing diseases
C. early deaths
B. cancer patients
D. injured men
Question 42: Who are more likely to have lung cancer and lung problems?
A. People who live with non - smokers.

C
Cau 1: Dap an A - tour

(ii.,, .

B. People who live in the country.
C. People who live with smokers.
D. People who live in the city.
Question 43: The main cause of mouth cancer, tongue cancer and throat cancer
is
A. overeating
B. smoking
C. breathing
D. drinking
Question 44: The number one cancer among men is
A. tongue cancer

B. lung cancer


C. mouth cancer

Question 45: The word "it" in the passage refers to
A. smoke
B. air
C. cancer

D. throat Qancer
D. breath

Read the following passage and mark the letter A , B, C , or D on your
answer sheet to indicate the correct word for each of the blanks from 46 to 50.
The agricultural revolution in the nineteenth century ...(46)... two things: the
invention of labor - saving machinery and the development of scientific agriculture.
Labour - saving machinery naturally appeared first where labor was scarce. "In
Europe", said Thomas Jefferson, "the object is to make the most of their land, labor
being abundant; here it is to make the most of our labour, land being abundant." It
was in America, therefore, that the great ...(47) in nineteen - century agricultural
machinery first came.
_.(48)... the opening of the century, with the exception of a crude plow, farmers
could have carried practically all of the existing agricultural implements on their
backs; by 1860, most of the machinery in use today had been designed in an early
form. The most important of the early ...(49)... was the iron plow. As early as 1790
Charles Newbold of New Jersey had been working on the idea of a cast - iron plow
and spent his entire fortune in introducing invention. The farmers, however, would
have none of it, claiming that the iron poisoned the soil and made the weeds grow.
Nevertheless, many people devoted their attention ...(50)... the plow, until in 1869
James Oliver of South Bend, Indiana, turned out the first chilled - steel plow.
Question 46:


A. refer

B. involved

C. relate

D. involving

Question 47:

A. advantage

B. advances

C. advanced

D. profit

Question 48:

A. At

B. on

C. in

D. From

Question 49:


A. inventor

B. distribution C. creation

D. inventions

Question 50:

A. to

B. for

D. at

130

C. in

H L T O N G DAN G I A I C H I T I E T

V i ta c6: /ou/ trong tour dupe phat am la /ua/. /ou/ trong cac tir: account,
sound, hour dugc phat am la /au/.
Cau 2: Dap an D - sight
Vi ta c6: /s/ trong sight du-gc phat am la /s/. /s/ trong cac tir con lai: music,
present, reason dugc phat am la /zf.
Cau 3: Dap an A - translated
Vi ta c6: /ed/ trong translated dugc phat am la / i d / , /ed/ trong cac tu con lai;
washed, laughed, watched dugc phat am la /t/.
Cau 4: Dap an D - throw
'

Vi ta c6: /th/ trong throw dugc phat am la /9/. /th/ trong cac tu con lai: father,
brother, mother dugc phat am la /3/.
Cau 5: Dap an B - skin
Vi ta c6: /i/ trong skin phat am la / i / . /i/ trong cac tu con lai: idea, find, flight
dugc phat am la/a I / .
Cau 6: Dap an B - not to make

!> ' t i a!

+ Ve mat ngu nghTa: "5o bdo loikhongdime lam onctitranhlamphien ba ta"
Ta C O cum tir bat djnh: to make noise: lam on

+ Ve mat ngij phap: Cau van de cap den cau true:
,; y:; ^
tell S.o (not) to do S.th: bao/ra lenh cho ai (khong) duoc lam gi
B la dap an dung duy nhat.
Cau 7: Dap an B - to
+ Ve mat ngu nghTa: "Zi? tre la quan trong nhat doi vai toi."
Ta C O cum tu: be important to S.o: quan trgng d6i vai ai
C la lira chon duy nhat.
Cau 8: Dap an D - delay
_ + Ve mat ngu nghTa: "Dieu quan trgng la chimg ta phai hoan cu()c hop cho
^^n ngay mai.'''
V U M .f»";+ Vg mat ngu phap: Quan sat cau van da cho ta thay c6 It is important that
^ay la hinh tlu'rc ciia dang gia djnh trong menh de danh tu. Ta da h\k It is
"Important that + S + Vnguyen v?n (ap dung cho tat ca cac loai chii ngu). Vay D la
'ya chon duy nhat dung.
Cac phu-ong an con lai khong phu ho-p vi:
A. delayed: thi qua khu don
Jfcjp. will delay: thi tuang lai don




,:t,':t"t^r'i^-'^-

must delay: dang cua must + Vnguyen v?n: phai lam gi

' '


Cau 9: Dap an D - to working
+ Ve mat ngu ngliTa: ''Chung toi dang dan quen vai Ijch lam viec 6 ngdy trong
mot tudn."
Ta CO cym: be accustomed to doing S.th: quen vol viec lam gi
Cau 10: Dap an A - are
+ Ve mat ngfr nghTa: "Ccinh sat dang truy lung ga dan 6ng da ddnh cdp chiec d
to."
+ Ve mat nm~r phap: Khi The police lam churc nang chu ngiJ trong cau thi dong
tir chia a hinh thirc so nhilu. Dya vao cac phuong an dap an da cho, ta nhanh
chong chon dircTC dap an A.
j^^^
Cau 11: Dap an D - A and B are correct
+ Ve mat ngCr ngiiTa: "//a hoi toi rdng li?u toi da Idm xong bdi tap ve nha

... too + adj/adv + (for S.o) + V to-inf:
qua
n6i (doi v6i ai) khong the lam gi
Cau 14: Dap an B - to being
+ Ve mat ngu nghTa: "Phu nu My quen vai sir tu do.'
be (get) used to + V-ing: quen vai viec lam gi


Cau 15: Dap an C - overweight
Xet cac dap an:

Dua vao ngu nghTa cau van, C la lira chgn duy nhat.
Cau 16: Dap an D. take care of
Xet cac dap an:

•SS^i^"
^Si

A. believe in (S.th): tin tuang vao (dieu gi)
B. bring about: gay ra
C. tear (S.th) off: ,\ toac (cai gi) ra
D. take care of (S.th): cham soc (cai gi)
Dua vao ngu nghTa cau van, ta chgn D la dap an dung.

+ Vg mat ngu ntzhTa: 'Sinig ngdy mai canphong cua toi se duac quet voi."
+ Ve mat ngu phap: Cau van d^ cap den cau true the nha bao:
T h i chu dgng: S + have + 0 1 (nguoi) + V nguyen ven + 0 2 (vat)
T h i bi dong: S + have + 02 (vat) + V3/ed: ai do c6 cai gi dugc lam

+ Ve mat ngfr nghTa: '•'Lam an trong nam moi thi't cdn than khi toi vdng nha."
Cau 17: Dap an C - Others

Vay A la lira chgn duy nhat.
Cac dap an con lai khong phu hgp.
Cau 13: Dap an D - so
+ Ve mat ngfr ngiua: "Bo phim qua hap dan den noi toi xem di xem Igi nhieii
Idn."

+ Ve mat ngu phap: Quan sat cau de ta thay
interesting that
Cau van
the hien dang thuc cua cau true:
: qua., den noi

D la lira ciign duy nhat diing.
Cac dap an con iai khong phu hop vi:
A. such: ta c6 cau true tuang dong nghTa vd\u true da d l cap tren:

m

^kh c.

+ Ve mat ngu nghTa: "Co dy an qua nhieu den ndi bay gia c6 dy dSi mat vai
viec thiea can."

Cau 12: Dap an A - have my room painted

C. too: ta CO cau true:

iw

B. weight (n): can nang
D. being weight (khong ton tai dang thuc nay)

B. very: ta c6: very + adj/adv: rSt

'H\A Oft uou


A. weigh (v): can

+ V^ mat ngjj phap: Quan sat cau de ta thay They asked me ... Cau van da cho
dang de cap den cau tuong thuat gian ti§p dira tren cau tuang thuat tryc tiep la mot
cau hoi bat d'au bSng mot trg dong tir a thi qua khu dan. Nhu ta biet, khi chuyen
doi cau hoi tr^rc tiep bat dau bang mot trg dgng tCr sang cau tuang thuat gian tiep
thi ta piiai dung chiJ "whether hoac i f de bat dau cau gian tiep. Vay D la dap an
duy nhat dung.

such + a/an + N + that +

ii'i:}

B la dap an duy nhat diing.

C. overweight (n): thua can

/ V

:.v':

+ Ve mat ngu phap: Cau de c6 dang cua cau true:

chuar

so + adj/adv + that +

den

: qua


den noi

+ Ve mat ngu phap: Cau van dang de cap den cau true Some....Others....:s6
nay
So kia....
Cac dap an con lai khong phij hgp.
A. Another + danh tu dem dirge so it
B. The others: nhung cai (nguoi) con lai

,

D. The other + danh tu dem dugc so it, ham y cai con lai
+ Ve mat ngu nghTa: "Hang nam, nhieu nguai ddn a cac mien que den thanh
Pho Ho Chi Minh de tim cong vi('c. Nhieu nguai trong sd do a dg tudi thanh thieu
^icn. Sd con lai la nguai km."
Cau 18: Dap an D-better
+ Ve mat ngu nghTa: "Anh ay trinh hay bdi llniyet trinh tot hori chung toi mong dai."
+ Ve mat ngu phap: Ta quan sat thay
than
Cau van the hien dang thuc
so sanh hon. Ta loai nhanh chong cac dap an A: well (adv), B: good (adj) va C:
fluently (adv) vi khong c6 hinh tiiiVc so sanh.

I

133


n h a n h CDBT TN_Anh VSn - Mg6_Van M i n h


Gih'i

Cty TimFTWT? DWH Knang vigt

Vay D la lua choii duy nhat (better la dang so sanh han cua well va good).
C a u 19: Dap an D - Neither

I f clause (S + V2/ed +

+ Ve mat ngCr nghTa: ''Khong phai ban ciia toi ma cung khong phai toi dang
khat mr&c."
+ Ve mat ngij phap: Quan sat cau van da cho ta thay
dang ciia cau true:

nor

) , main clause (S + would + Vnguyen ven): dien ta mot

dieu trai hSn vai thvrc te hign tai.

Cau van c6

C la dap an duy nhat diing.
Cac dap an con lai khong phu hop vi:

.*«*;ci j,.,.

v


A. w i l l buy: w i l l + Vnguyen ven dugc dung trong m?nh de chinh ciia cau dieu
kien loai I .

Neither + N1 + nor + N2 + V 2 (N2): Khong

ma cung khong

C. had bought: had + V3/ed dugc diing trong menh de I f ciia cau dieu kien loai I I I .

Vay D la phuong an duy nhat diing.

D. am buying: thi hien tai tiep dien, khong diing trong cau dieu kien.

Cac dap an con lai khong phii hop.

u 24: Dap an D - suggested

C a u 20: Dap an A - took
+

Ve mat ngir nghTa: ""Toi cia mdt hai tichig dongho

+

Ve mat ngij phap: Ta c6 cau true:

cfSsira nidi nha."

' .


+

Ve mat ngfr nghTa: "Toi de nghi ban nen di khdm bdc sTsam nhdl c6 the."

*,<'"

+

Ve mat ngCr phap: Ta c6 cau true:

>

SI + suggest + S2 + (should) + V ngiiyen ven: ai do khuyen ai nen lam gi
Cau triic dong y nghTa: S2 + had better + Vnguyen ven = It's (high) time S2

It takes S.o + khoang thoi gian + V to inf: A i ton thoi gian de lam gi

V2/ed: ai nen lam gi

A la dap an duy nhat diing.
+

Ve mat ngir nghTa: "Toi ihich chup dnh."

+

Ve mat nizu phap: Quan sat cau de ta thay /

i


D la dap an duy nhat diing.

Cau 21: Dap an A - enjoy talking

Cac dap an con lai khong phii hgp.
photographs.

Dira vao cac

'•'

M o rong:

plurong an dap an da cho, ta loai ngay dap an B: to take vi ta khong dijng V t o inf

-

diing sau chu ngir, C: don't mind taking a vi "a" khong di v 6 i danh tir so nhieu

Eg: The judge ordered that the prisoner should be remanded: Quan toa ra l?nh

(photographs), D: hate taking one vi "one" cung nhu "a" khong di vai danh tir so
nhieu (photographs). Vay A la dap an diing duy nhat.

tii nhan phai tra lai trai tam giain de dieu tra them.
-

Regret + V-ing: tiec da lam gi

Eg: I regretted talking to you the truth: Toi lay lam tiec da noi cho ban nghe su that.


enjoy + V-ing: thich lam gi

-

C a u 22: Dap an D - has been
+ Ve mat ngir nghTa: "So vu tai nan giao thong clir&ng bp khong ngitng
tang trong nhiing nam vira qua."'
+

SI + order + that + 82 + should + V nguyen ven: ra lenh cho ai lam gi

gia

Ve mat nuiT phap: Ta quan sat thay cum " i n the last few years: trong nhirng

nam viia qua" trong cau van da cho. Day la cum d i l n ngiJ chi thai gian sCr dung
trong thi hi$n tai hoan thanh. Ta loai nhanh chong dap an B: are (thi hien tai don)
va B: w i l l have been (thi tirang lai hoan thanh). Hon niTa, ta lai thay cau van sir
dung chii ngij gia There .... A n . . . . : ta phai dung danh tir dem dugc so it truac A n .

Regret + V to-inf: tiec khi lam gi

Eg: I regret to tell you that your application was turned down: T o i lay lam tiec
khi noi v 6 i ban rang don xin vi^c ciia ban khong dugc chap nhan.
-

T '>

inform S.o of/about S.th: noi cho ai biet ve cai gi


Eg: He informs me about Mary: A n h l y noi cho toi biet ve Mary.
Cau 25: Dap an C - neither do I
+

Ve mat ngiT nghTa: "Anh ay khong thich xem tranh vd toi cung vay."

+

V § mat ngir phap: Ta quan sat thay He doesn't like... trong cau van da cho.

Do do, ta loai tiep tuc dap an A : have been.

Cau van su dung thi hien tai don a the phii djnh. Quan sat cac dap an da cho, ta loai

'

ngay dap an A : so am I va D: I am, too vi "so va too" chi dugc dung trong cau khapg

Vay D la dap an duy nhat diing.

djnh, loai B: neither did I v i " d i d " la trg dgng tir diing a thi qua khii- don. Vay C la

Cau 23: Dap an B - would buy
+ Ve mat ngu nghTa: "Neti toi c6 dii ticm, toi se mua cuon sdch ddy du thong tin
hiru dung do.'"
+

Ve mat ngiJ phap: Quan sat cau van da cho ta thay If I had


Day la hinh

thuc trong menh de gia sir ciia cau dieu ki?n loai I I . Cau de dang hoi dgng tir trong
menh de chinh ciia cau dieu kien loai I I :
134

dap an duy nhat diing vi Neither dugc diing de dap lai y phii djnh nhu ve triiac.
u...

Cau 26: Dap an C - It's good
+

Dich nghTa: "Cuon sach nhu the nao? - N o hay iSm."

Cau de sir dung tii' How: nhu the nao. Cau tra lai tuong ung trong truong hgp
nay phai la mot tinh tir mieu ta vat dang dugc hoi.

... ^
135


^

TJtarnTianTi UDBT TM Anh van - Ngo van iviinn

Xet cac dap an:

A. It's expensive: No dat lam. (Dung khi hoi vS gia ca cua cu6n sach)
B. It's written in English: No viet bang tieng Anh. (Dung khi hoi cuon sach do
dugc viet bang ngon ngO gi)

C. It's good: No hay lam. (Dung khi hoi cuon sach nhu the nao)
D. They're old: Chung cQ roi. (Diing khi hoi cuon sach dugc viet khi nao. Dua vao
ngiJ ngliTa cua cau hoi va cac phuong an dap an, ta chgn C la dap an diing duy nhat.
Cau 27: Dap an D. Yes. There are two by the door.
+ Ve mat ngu nghTa: "Con cho nao trong khong? - "Da c6. Con hai cho canh
cua so."
Cac phuong an khac khong phu hop:
A. Tat ca cac ban deu chat kin.
B. Khong CO ai a day ca.
C. Cai ban nay can dugc son.
Cau 28: Dap an A - Yes, please. Orange juice is fine.
+ Dich nghTa: "To/ mang cho ban do iiong nhe iihe?- "Vang. Nicac cam thi
dime. "
Xet nghTa cac dap an:
(
B. It's very kind of you: Ban that tu te, chua day du thong tin.
do C. Ok. You can: Dugc. Ban c6 the, ta khong diing each dap lai nay.
D. 1 feel quite fine: Toi cam thay kha tot.
Cau 29: Dap an B - Thanks
+ Dich nghTa: ''Toi dongy v&i ke hoach cm ban."
Xet nghTa cac dap an:
A. Oh, no: A, khong.
B. Thanks: Cam on.

ti,}'

C. Yes, you did: Vang, ban da thuc hien dugc.
D. It belongs to you: No ban thuoc quyen so huu cua ban.
Dua vao nghTa cua cac dap an, B la lua chon duy nhat.
Cau 30: Dap an C . so do 1

+ Ve mat ngu nghTa: "Anh ay thich xem phim tren tivi." - "Toi cung vay."
+ Ve mat ngir phap: Ta c6 each dap lai "...cung vay" trong y khang dinh cua
ve truoc la "So + trg tu + S". V i cau g6c da cho (he likes
) mang hinh thuc
khang djnh a thi hien don. Do do, ta chgn C . so do I la dap an dung duy nhat. Ta
loai cac phuong an khac vi khong hgp ngQ- phap.
Cau 31: Dap an C - but she is better now
+ Ve mat ngu nghTa: "'Paula da bi dm hai thang vua qua nhung bay gia cd dy
khoe hem roi.'"
Xet cac dap an:
136

uiy I iMHH MTV D W H KFiang vigi

A. she is working hard now: bay gia c6 ay dang lam viec cham chi, cau van
thi^u lien tii- noi giua hai menh de.
B. although she is taking the medicine: mac du c6 iy dang uong thuoc.
/although CO the dung dau cau hoac dung giua cau lam lien tir noi, truac va sau no
Ichong CO dau cham cau.
C. but she is better now: nhung bay gia c6 ay khoe hon roi ("but" dugc dung de
(jiln ta y trai ngugc nhau, thuong dung sau dau phay va bat dau cho menh d^ thu hai)
D. because she stayed up late last nigiU: bai vi c6 ay thuc khuya dem qua.
f ruoc va sau because khong c6 dau cham cau.
^ .
CSu 32: Dap an D - the government will go ahead with it
+ Ve mat ngir nghTa: "Neu die an cd lien qucm den ngimi dan thi chinh phu se
cho tien hdnh.''
+ Ve mat ngu phap: Ta quan sat thay ve dau cau van c6 dang cua cau dieu
ki?n. Dong tu a menh de I f clause chia thi hien tai don. Do do, cau van c6 dang ciia
cau dieu kien loai I :

If clause (thi hien tai don), main clause (thi thuang lai don)

'
'

;j

D la dap an duy nhat dung.
Cac dap an con lai khong phii hgp vi:
A. they would have gone ahead with it, dCing cho m^nh de chinh cua cau dieu
kien loai I I I .
B. he had to solve the problem, la mot menh de dgc lap, khong dugc dung trong
cau dieu kien.
C. they are very happy to know that, khong dung dang menh de nay trong menh
de chinh cua cau dieu kien loai I .
D. the government will go ahead with it, dung trong menh de chinh cua cau dieu
kifn loai I .
'
D la dap an duy nhat dung.
Cau 33: Dap an A - He told as i f
+ Ve mat ngu' nghTa: "Anh dy noi nhu the la toi horn qua anh dy da gap toi."
+ Ve mat ngfr phap: Ta quan sat thay menh de trong cau da cho c6 dgng tu
chinh chia thi qua khu hoan thanh. Dua vao cac phuong an dap an da cho, ta loai
nhanh chong dap an C: They suggested that vi ta c6:
S1 + suggest + S2 + (should) + V nguyen vgn: ai do khuyen ai nen lam gi
Ta loai tiep dap an D: I f he had been there vi day la menh de I f ciia cau dieu
loai I I I , menh de chinh theo sau c6 dang: S + would/could + have V3/ed.
Nhu ta bilt. As if + qua khur hoan thanh: dien ta dieu khong c6 that trong qua
V^y A la dap an duy nhat dung.


* «! •

:Jti;V''-:s'\t.
m


Cty TNHH MTV DVVH Khang Vi§l

G\i\h CDBT TN Anh VSn - N96 Van Minh

Cau 34: Dap an A - as they walk
+ Dich nahTa: 'T/??/ ml- c6 the ctoi x6 nuac tren cfau khi ho di ho"
Xet cac dap an:
A. as they walk: khi ho di bo, dung ve ngu phap Ian ngCr ngliTa.
B. so as to be enough for, khong du thanh phan cau.
C. when they walked: khi ho di bo, khong thich hop cho cau van tren vi menl,
de dung truoc sir dung "can".
D. while they were walking: trong khi ho dang di bo, sal ve mat ngiJ phap (su
dung thi qua khu ti^p dien)
Cau 35: Dap an B - heavy for me to carry
+ Ve mat ngu nghTa: ''Xd mmc qua ngng clin noi toi khong the xdch no."
+ Ve mat ngu phap: Quan sat cau van da cho ta thay c6
too .... va dua vao
cac phuong an dap an da cho, ta biet dugc cau van c6 dang c6 dang ciia cau true:
.... too adj/adv (for S.o) to do S.th: qua .... (doi voi ai) khong the lam gi
B la Kra chon duy nhat.
Lu'u y: Voi cau true .... too ....to

ta khong dung tan ngu cuoi cau neu tan


ngir va chii ngu cua cau la mot doi tugng.
Cau 36: Dap an B - come
comes
+ Ve mat ngu nghTa: "//aw hit luang nuac chung ta su dung deu c6 ngudn goc
tirsong, ho va bau khiquyen."
+ Ve mat ngu phap: Come la dong Xvt chinh cua cau van da cho. Va y nghTa
cua cau van de cap din su that {mt&c chung ta su dung deu c6 ngudn goc tie son^,
hS va bau khi quyin.) nen ta dung thi hien tai don.
Cau 37: Dap an A - These
The
+ Ve mat ngiT nghTa: ''Bdc sT da kiiin tra hiiyet dp cua toi trong khi kiem tni
sue khoe."
Ta khong the diing These vi doctor la danh tu s6 it.
Cau 38: Dap an B - with

for

+ Vh mat ngiJ nghTa: ''Ban khong thi do iSi cho nhung chuyen da xdy ra vi anh
ay khong c6 mat a day toi horn qua.'"
Ta CO cum: blame for S.th: do loi cho cai gi
Cau 39: Dap an C - is —> were
+ Ve mat ngu nghTa: "Co dy luon tro chuyen vai dua be nhu the dua be la im'l
ngirai Idn."
'*
+ Ve mat ngij phap: Ta quan sat thay cau van su dung....as though..... Nhu ta
biet, as though/as if+ thi qud khu dan/qua khu hodn thanh. Ta xet thSy ye dau caU
van su dung thi hi?n tai don nen menh de trang ngu chi the each bk dau bSng ^'^
though theo sau phai dung thi qua khu don de dien ta dieu khong c6 that a hien ta'
Cau 40: Dap an A - stay

138

stayed

+ Ve mat npfr nghTa: "Afe nnidn con a nhd hern la di da ngogi."
+ Ve mat ngu phap: Ta c6 cau true:
S j j - would rather + S2 + V2/ed + than + V2/ed: ai do thich ai lam gi hon lam gi
* Bai djch de nghi:
Hut thuoc la chinh la nguyen nhan gay ra benh ung thu phoi, mpt can b?nh pho
{jjin nhat doi voi dan ong. Theo u6c tinh, ti le tu vong cua nhCrng nguoi mSc can
bfnh nay la 90%. Viec hut thuoc la ciing gay ra cac benh ung thu ve mieng, luoi va
c6 hong. Nhung nguoi hut thuoc la thuong m3c benh tim va viem phoi. Viec hut
thuoc la cuop di sinh mang gAn mot trieu nguoi tren toan the gioi hang nam. Hut
thuoc la khong nhung gay hai cho ban than nguoi hut thuoc ma con gay hai cho
nh&ng nguai xung quanh. Klioi thuoc thuong duoc thai ra trong bau khi chung ta
hit tho. Con tre, vg hoac chong c6 the hit khoi thu6c tu nhung nguoi hut thu6c.
Nhfl-ng tre em song trong gia dinh c6 bo me hut thu6c thuong mSc cac van de ve
hoi tha va phoi hon la nhung dua tre khac. Ti le mSc benh ung thu phoi cua nhung
phu nu ket hon voi dan ong hut thuoc thuong cao hon nhung phu nu kk hon voi
dan ong khong hut thuoc.
Con nguoi hau nhu y thuc dugc tac hai cua viec hut thuoc d6i voi sue khoe cua
hp. Vay thi tai sao ho v i n hut thuoc?
Cau 41: Dap an C - early deaths (doan 2)
Viec hut thuoc la cuop di sinh mang gan mot trieu nguai tren toan the gioi hSng nam.
Cau 42: Dap an C - People who live with smokers (doan 4)
Doi tugng nao chiem ti le cao mSc b?nh ung thu ph6i va cac can benh ung thu
khac? - nhOng nguai song chung voi nguoi hut thuoc.
j
Cau 43: Dap an B - smoking (doan 1)
Nguyen nhan chinh gay ra cac can benh ung thu ve mieng, luoi va c6 hong la

vi?c hut thuoc la.
Cau 44: Dap an B - lung cancer (doan 1)

,

Can benh pho bien nhat doi voi dan ong la benh ung thu phoi.
CSu 45: Dap an A - smoke
Tir "it" trong doan van de cap den viec hut thudc.
* Bai dich de nghj:
Cuoc each mang nong nghiep a t h i ky 19 g6m hai ngi dung ca ban do la viec
Pijat minh ra may moc tiet kiem sue lao dong va viec phat triln khoa hoc trong
Ijong nghiep. May moc tiet ki?m sue lao dong ra dai ISn dAu tien tai nhOng nai
'^u hut lao dong. Thomas Jefferson da tung noi r5ng, a chau Au, nhung thiet bj
'^U'o'c tao ra nham miic dich tan dung toi da ngu6n tai nguyen dat phi nhieu va
| n nhan cong doi dao con a day thi nhung may moc do dugc tao ra eung nham
' 'Jyng toi da ngudn tai nguyen dat phi nhieu va ngu6n nhan cong d6i dao. Do do,


•Giai nhanh CUBTTW mn van - wgo van mnrn-

,

_^

nhung tien bo vupt bac trong viec sir dung may moc phuc vy nong nghi?p da xuat

oLy i i M n n rarv u v v n Miang vi?i

pHANBON


hi?n \kn dau tien chinh tai nuoc M y vao the ky 19.
Vao giai doan dau ciia th§ ky nay, ngoai t r u chiec cay tho so thi nguai nong dan

G I A I C A C D E THI T6T N G H I E P THPT V A TUYEN S I N H

phai mang tat ca nhiJng dung cy d6ng ang thai bay gia tren doi vai. M a i den nam

OAI H O C , C A O D A N G C U A B O G I A O D U C

1860, dua tren nhung thiet ke ban dau, nguoi ta da sang c h l ra cac may moc sir

V A O A O T A O lU N A M 2 0 0 6 D E N 2011

dung ngay nay. Phat minh quan trong nhat thai bay gia la chiec cay bang sat. Vao
dL

nam 1790, Charles Newbold d i n tir bang New Jersey da thvrc hien y tuoiig

sang tao chiec cay sfit va ong da danh toan bo thai gian cua ong de cho ra doi phat

DE

THI

T U Y E N

S I N H

minh nay. Tuy iihien, nguai nong dan lai khong he quan tam den y tuong nay vi ho


D A I

MA

H O C K H O I

D E :

D

N A M

2 0 1 1

3 6 9

cho rSng cai cay b5ng sSt gay hai cho dat dong thai lam cho c6 dai moc ram hon.
N h u n g cimg c6 nhieu nguai that su quan tam tai no. Cho den nam 1869, James
Oliver den t u viing South Bend, bang Indiana da thiet ke lai no thanh mot chiec cay
bang s3t lanh.
C a u 46: Diip an B - involved

T H I G O M 80 C A U ( T l T Q U E S T I O N 1 D E N Q U E S T I O N 80)
Mark the letter A, B, C or D on your answer sheet to indicate the word or
phrase that is closest in meaning to the underlined part in each of the following
questions.
Question 1: We have lived there for years and grown fond o f the surroundings.
That is why we do not want to leave.

Xet cac dap an:

A . refer (to): de cap van de gi
B. involved (v): bao gom, lien quan
C. relate (to): lien quan den

A . planted many trees in the surroundings

*\

B. haunted by the surroundings

'*

C. loved the surroundings

D. involving ( V - i n g cua involve): lien quan, khong hop thi

D. possessed by the surroundings

=> B la dap an dung duy nhat.

':

Question 2: His new work has enjoyed a very good review from critics and readers

C a u 47: Dap an B - advances

A. viewing

Xet cac dap an:
A . advantage (n): sir thuan Igi


B. advances (n): su tien bo

C. advanced (adj): cao cap, tien bp

D . profit (n): Igi nhuan

B. regard

C. look

D . opinion

Question 3: Such problems as haste and inexperience are a universal feature o f youth
A. marked

B. shared

part that needs correction in each of the following

C a u 48: Dap an A - A t
Vao thai gian dau ciia khoang thai gian nao do, ta dung " A t tiie beginning o f
C a u 49: Dap an D - inventions

A

B

C


recent regional book fairs.
D

A . inventor (n): nguai phat minh

Question 5: During our tour o f the refiney. it was seen that both propane and

B. distribution (n): sir dong gop

A

C. creation (n): viec giai tri, tro tieu khien

B

C

.

gasoline were produced in !ar^e volumes

D. inventions (n): su phat minh

D
Question 6: The first important requirements for you to become a mountain

=> D la dap an duy nhat dung.
" ' j !

A


B

^''mber are your strong passion and you have good health

, ,„ s

0
. ^
D
^"estion 7: Hardly did he enter the room when all the lighets went out.
A
140

underlined

questions.

Question 4: Publishing in the U K , the book has won a number of awards m

Xet cac dap an:

• ' Ta c6: devote S.th to S.th: cong hien cai gi cho cai gi

D . separated

Mark the letter A, B, C, or D on your answer sheet to show the

=> B la lira chon duy nhat.


C a u 50: Dap an A - to

C. hidden

B

C



,
,
, >

D



• fil


Question 8: A professor o f economy and history at our university develped a new
A

B

theory o f the relationship between historical events and financial crises.
C

D


Read the followiii'fi passage adapted from Understanding R u r a l America ^
InfoUSA and mark the letter A, B, C, or D on your answer sheet to indicate tl,^

{ark the letter A, B, C, or D on your answer sheet to indicate the word or
Ijirase that is O P P O S I T E in meaning to the underlined part in each of the
following questions.
^„ .
Question 19: Fruit and vegetables grew in abundance on the island. The islanders
even exported the surplus.
A. sufficiency

B. excess

C. large quantity

D. small quantity

Question 20: There is growing concern about the way man has destroyed the

correct word for each of the blanks from 9 to 18.

environment.
The well-being o f America's rural people and places depends upon many things
-

the availability o f good-paying jobs; (9)

to critical services such as


A. attraction

.
B . speculation

C. ease

,

D . consideration

Mark the letter A, B, C, or D on your answer sheet to indicate the

sentence

education, health care, and communication; strong communities; and a healthy

that is closest in meaning to each of the following

natural environment. A n d , (10)

Question 21: " D o n ' t forget to tidy up the final draft before submission," the team

urban America is equally dependent upon

questions.

these things, the challenges to well-being look very different in rural areas than in

leader told us.


urban areas. Small-scale, low-density settlement (11)

A. The team leader asked us to tidy up the final draft before submission.

make it more costly

for communities and businesses to provide critical services. Declining jobs and
income in the natural resource-based industries that many rural areas depend on
(12)

'

workers in those industries to find new ways to make a living. Low-

skill, low-wage rural manufacturing industries must find new ways to challenge the
increasing number o f (13)

competitors. Distance and remoteness impede

many rural areas from being connected to the urban centers o f economic activity.
Finally, changes in the availability and use o f natural resources located in rural
areas (14)
who (15)

the people who earn a living from those resources and thosc
recreational and other benefits from them.

Some rural areas have met these challenges successfully, achieved some level
o f prosperity, and are ready (16)


the challenges o f the future. Others have

neither met the current challenges nor positioned themselves for the future. Thus,
concern for rural America is real. A n d , while rural America is a producer o f critical
goods and services, the (17)

goes beyond economics. Rural America is

also home to a fifth o f the Nation's people, keeper o f natural amenities and national
treasures, and safeguard o f a/an (18)

part o f American culture, tradition,

B. The team leader reminded us to tidy up the final draft before submission.
C. The team leader ordered us to tidy up the final draft before submission.

^ '

D. The team leader simply wanted us to tidy up the final draft before submission
Question 22: " M y company makes a large profit eveiy year. Why don't you invest
more money in it?" my friend said to me.
A. M y friend suggested his investing more money in his company.
B. M y friend persuaded me to invest more money in his company.

,

J C. I was asked to invest more money in my friend's company.

j^j"* '


D. M y friend instructed me how to put more money into his company. ^ , ',^|^',
Question 23: " M u m , please don't tell dad about my mistake," the boy said.
,i,:A. The mother was forced to keep her son's mistake as a secret when he insisted
^ i p . The boy earnestly insisted that his mother tell his father about his mistake.
JFI C. The boy begged his mother not to tell his father about his mistake.
D. The boy requested his mother not to talk about his mistake any more.
Question 24: " Y o u shouldn't have leaked our confidential report to the press,
Frank!" said Jane.

and history.
C.access

D. advantage

Question 9:

A. challenge

B. key

Question 10:

A. because

B. while

C. when

D. since

D. pattefns

Question 11:

A. styles

B. tools

C. means

Question 12:

A. offer

B. turn

C. force

D. make

Question 13;

A. foreign

B. abroad

C. lateral

D. rural


Question 14:

A. effect

B. encourage

C. stimulate

D. affect

Question 15:

A. involve

B. evolve

C. bring

D. derive

Question 16:

A. in

C. with

D. for

Question 17:


A. research

B.of
B. impatience

C. concern

D. stimulus

B. simple

C. incredible

D. unique

Question 18:

>j; '

A abnormal

A. Jane accused Frank o f having cheated the press with their confidential report.
i B . Jane criticized Frank for having disclosed their confidential report to the press
* C . Jane suspected that Frank had leaked their confidential report to the press.
' D. Jane blamed Frank for having flattered the press with their confidential report
Question 25: " I f you don't pay the ransom, w e ' l l kill your boy," the kidnappers
told us.
A. The kidnappers pledged to k i l l our boy i f we did not pay the ransom.

'


B. The kidnappers ordered to kill our boy i f we did not pay the ransom.
C. The kidnappe rs threatened to kill our boy i f we refused to pay the ransom.
D. The kidnappers promised to kill our boy i f we refused to pay the ransom. '
.


v^ty inn-nTTvrTv u v v n ^^a^q

* Read the following passage adaptedfrom Cultural Guide - O A L D , and mark
the letter A, B, C, or D on your answer sheet to indicate the correct answer to
each of the questions from 26 to 35.
The issue of equality for women in British society first attracted national
attention in the early 20"' century, when the suffragettes won for women the rigi^
to vote. In the 1960s feminism became the subject of intense debate when the
women's liberation movement encouraged women to reject their traditional
supporting role and to demand equal status and equal rights with men in areas such
as employment and pay.
Since then, the gender gap between the sexes has been reduced. The Equal Pay
Act of 1970, for instance, made it illegal for women to be paid less than men for
doing the same work, and in 1975 the Sex Discrimination Act aimed to prevent
either sex having an unfair advantage when applying for jobs, hi the same year the
Equal Opportunities Commission was set up to help people claim their rights to
equal treatment and to publish research and statistics to show where improvements
in opportunities for women need to be made. Women now have much better
employment opportunities, though they still tend to get less well-paid jobs than
men, and very few are appointed to top jobs in industry.
In the US the movement that is often called the "first wave of feminism" began
in the mid 1800s. Susan B. Anthony worked for the right to vote, Margaret Sanger
wanted to provide women with the means of contraception so that they could

decide whether or not to have children, and Elizabeth Blackwell, who had to fight
for the chance to become a doctor, wanted women to have greater opportunities to
study. Many feminists were interested in other social issues.
The second wave of feminism began in the 1960s. Women like Betty Friedan
and Gloria Steinem became associated with the fight to get equal rights and
opportunities for women under the law. An important issue was the Equal Rights
Amendment (ERA), which was intended to change the Constitution. Although the
ERA was not passed, there was progress in other areas. It became illegal for
employers, schools, clubs, etc. to discriminate against women. But women still find
it hard to advance beyond a certain point in their careers, the so-called glass ceiling
that prevents them from having high-level jobs. Many women also face the
problem of the second shift, i.e. the household chores.
In the 1980s, feminism became less popular in the US and there was k'SS
interest in solving the remaining problems, such as the fact that most women sti"
earn much less than men. Although there is still discrimination, the principle that i'
should not exist is widely accepted.
Question 26: It can be inferred from paragraph 1 that in the 19"' century,
A. British women did not complete their traditional supporting role
B. most women did not wish to have equal status and equal rights
C. British women did not have the right to vote in political elections
D. suffragettes fought for the equal employment and equal pay.

Question 27: The phrase "gender gap" in paragraph 2 refers to
A. the visible space between men and women
n the difference in status between men and women
C. the social distance between the two sexes
D. the social relationship between the two sexes

^


Question 28: Susan B. Anthony, Margaret Sanger, and Elizabeth Blackwell are
mentioned as
.
A. American women who were more successtul than men

B. American women with exceptional abilities
n
C. pioneers in the fight for American women's rights
D. American women who had greater opportunities
Question 29: The Equal Rights Amendment (ERA)
.
A. was not officially approved
mn.'!*-, •
B. changed the US Constitution
'
C. was brought into force in the 1960s
D. supported employers, schools and clubs
Question 30: In the late 20" century, some information about feminism in Britain
was issued by
.
A. the Equal Rights Amendment
B. the Equal Pay Act of 1970
C. the Equal Opportunities Commission
D. the Sex Discrimination Act
Question 31: Which of the following is true according to the passage?
'
A. The movement of feminism began in the US earlier than in Britain. .
B. The vvonicn's liberation movement in the world first began in Britain. '
C. The US movement of feminism became the most popular in the late 20''' century
D. The British government passed laws to support women in the early 20"' century

Question 32: The phrase "glass ceiling" in paragraph 4 mostly means
.
A. an imaginary barrier
B. an overlooked problem
C. a ceiling made of glass
D. a transparent frame
*
Question 33: Which of the following is NOT mentioned in the passage? ^
A. There is now no sex discrimination in Britain and in the US.
•*
B. Many American women still face the problem of household chores.
C. An American woman once had to fight for the chance to become a doctor.
D. British women now have much better employment opportunities.
Question 34: It can be inferred from the passage that
.
A. the belief that sex discrimination should not exist is not popular in the US
B. women in Britain and the US still fight for their equal status and equal rights
C. the British government did not approve of the women's liberation movem^it
t). women do not have better employment opportunities despite their great efl'orts
....

144

Vi^t


GiJi nhanh CDBT TN Anh VSn -

Vfln Minh


Cty TNHH MTV DVVH Khang Vi§t

Question 35: Whicii o f tlie following would be the best title for the passage?
A. Opportunities for Women Nowadays
B. Women and the Right to Vote

C. Failing to apply to that prestigious institution, his academic record at high
school was poor.

C. The Suffragettes in British Society
D. Feminism in Britain and the US
Mark the letter A, B, C, or D on your answer sheet to indicate the word that
differs front the rest in the position of the main stress in each of the following,
questions.
Question 36:

A . future

B. prospect

B. His academic record at high school was poor as a result o f his failure to
apply to that prestigious institution.

C. guidance

D . involve

Question 37:

A . facilitate


B. hydrology

C. participate

D. intimacy

Question 38:

A . represent

B. permanent

C. continent

D . sentiment

Question 39:

A . romantic

B. reduction

C. popular

D. financial

Question 40:

A optimist


B. immediate

C. fabulous

D. accuracy

Mark the letter A, B, C, or D on your answer sheet to indicate the sentence
that best combines each pair of sentences in the following

questions.

Question 41: He behaved in a very strange way. That surprised me a lot.

D. His academic record at high school was poor; as a result, he failed to apply
to that prestigious institution.
uestion 45: Smoking is an extremely harmful habit. Y o u should give it up
immediately.
i A. When you give up smoking immediately, you w i l l affect your health with
this harmful habit.
,
.
B. You should give up smoking immediately and you w i l l fall into an extremely
harmful habit.
C. Stop your smoking immediately so it w i l l become one o f your extremely
harmful habits.
D. As smoking is an
immediately.

extremely


harmful habit, you

answer to each of the following

B. I was almost not surprised by his strange behaviour.

Question 46: " W h y don't you sit down and

D. His behaviour was a very strange thing, that surprised me most.
Question 42: He cannot lend me the book now. He has not finished reading it yet,
A . Having finished reading the book, he cannot lend it to me.
B. He cannot lend me the book until he has finished reading it.
C. Not having finished reading the book, he w i l l lend it to me
D. As long as he cannot finish reading the book, he w i l l lend it to me.
Question 43: Crazianna is a big country. Unfortunately, it has never received
respect from its neighbours.
A . Crazianna has never received respect from its neighbours because it is a bia
country.
B. Crazianna is such a big countr>' that it has never received respect from its
neighbours.
C. It is Crazianna, a big country, that has never received respect from ii''
neighbours.
D. Though Crazianna is a big country, it has never received respect from il^
neighbours.
Question 44: His academic record at high school was poor. He failed to apply
that prestigious ins.titution.
A . His academic record at high school was poor because he didn't apply to tlu''
prestigious iuilitution.
146


give

it up

Mark the letter A, B, C, or D on your answer sheet to indicate the correct

A. He behaved very strangely, which surprised me very much.
C. What almost surprised me was the strange way he behaved.

should

questions.

A. make yourself at peace

j , . ^ ^ ^ .

C. make it your own home
Question 47: " Y o u
for lunch."

D. make yourself at home

B. oughtn't

C. needn't

Question 48: The Second World War
A . brought about

A. turn down

C. broke out

B. speed up

C. take up

D. put down

takes place varies widely from material to

.i

A . which melting

B . at which melting

C. at which they melt

D. which they melt

Question 51: The village was

visible through the dense fog.

B. barely

Question 52:


D . took out

i f we want to get there in time."

Question 50: The temperature

A. only

D . couldn't H)

in 1939.

B. turned up

Question 49: " W e ' d better

^:

B . make yourself at rest

have cooked so many dishes. There are only three o f us

A. wouldn't

material.

?"

C. mostly


D. hard

without animals and plants?

A. What would life on earth be like

B. How would life on earth be for

C. What w i l l life on earth be like

D. How w i l l life on earth be like

Question 53: Harry: " A r e you ready, Kate? There's not much time left."
Kate: "Yes, just a minute.
A . N o longer

B. 1 won't

finish

!"
C. I ' d be O K

D . I ' m coming
147


'G\i\h CDBT TN Anh Van - Ngfi van Minh

Question 67: The sky was cloudy and foggy. We went to the beach,

Question 54:

you treat him, he'll help you. He's so tolerant."

A. No matter how

B. In addition to

Question 55: I could not

C. Even though

D. As i f

the lecture at all. It was too difficult for me.

A. get along

B. make o f f

C. take in

D. hold on

Question 56: I did not want to believe them, but in fact,

was true

A . what they said


B. what has said

C. that they were said

D . which they said
a red hat."
D. is wearing

Question 58: Alfonso: " l had a really good time. Thanks for the lovely evening."
Maria: "

."
B . Yes, it's really good

C. Oh, that's right

D . N o , it's very kind o f you

Question 59: This shirt is
C. a bit less expensive

|, B. Shall 1 make vou like some coffee
C. Why don't we cook some coffee

. j , JQ ^g,

D. Would you mind making some coffee
A . us not chatting

.


during the working hours.

B. we didn't chat C. us not chat
p(issa>;e adapted front

D. we don't chat

A. Brim's' article

the term hroadcastiiiy,, from the treatment and care of the soil and of what grows
on it. It is directly related to cultivation and the adjectives ciilliiral

and ciil/urcd <\rc

part of the same verbal complex. A person of culture has identitlable attributes,
word culture does not refer solely to such knowledge and interest nor, indeed, to

C. Not completely

D. Why not?

Question 62: The instructor blew his whistle and

education. A t least from

19"' century onwards, under the influence o f

both in the singular and the plural {cultures)


B . o f f ran the runners

C. o f f were running the runners

D. the runners run o f f

Question 63: She built a high wall round her garden

the

anthropologists and sociologists, the word culture has come to be used generally

.

A. o f f the runners were running

to refer to a whole way of life of

people, including their customs, laws, conventions, and values.

ob

Distinctions have consequently been drawn between primitive and advanced
culture and cultures, between elite and popular culture, between popular and mass

.

culture, and most recently between national and global cultures. Distinctions have

A . to enable people not taking her fruit

B. so that her fruit would be stolen

been drawn too between culture and civiliziiHon;

*

C. to prevent her fruit from being stolen

the latter is a word derived not,

like culture or agriculture, from the soil, but from the city. The two words arc

\,

sometimes treated as synonymous. Yet this is misleading. While civilization and

D. in order that her fruit not be stolen
Question 64: Before I left for my summer camp, my mother told me to take warm

barbarism are pitted against each other in what seems to be a perpetual behavioural
pattern, the use of the word culture has been strongly influenced by conceptions o f

C. so that

Question 65: "Never be late for an interview,
C o r so

D . whereas

you can't get the j o b . "

D. i f not

for the heavy storm, the accident would not have happened.
B. hadn't been

,

among them a knowledge o f and interest in the arts, literature, and music. Yet the

B. I think that, too.

B. otherwise

^

Culture is a word in common use with complex meanings, and is derived, like

"

"

B. in case

,

D . much far expensive than

A. Yes, I ' m afraid not.

A. weren't


1 m sorry, but I can t do it now.

answer

Question 61: Sue: "Can you help me with my essay?"

A . despite

..i

Microsoft® Student 2008, and mark the letter A, B, C, or D on your

D . not to approach

it was cold.

, Mike?"

sheet to indicate the correct answer to each of the questions from 71 to HO.

C. not to enter

Question 66: I f it

luestion 69: .loan: "Our friends are coming.

^

B. not nearly as expensive as


B . not to smoke

A . unless

B. come on
D. come among

culture,

A. not to photograph

clothes with me

him and his weekend fishing trip.

on

Question 60: The sign "NO T R E S P A S S I N G " tells you
ripuMi/-

.

D. yet

A. come between

Read the following

that one.


A . as much expensive as

C. so

C. come up

Question 70: Our boss would rather

A . I ' m glad you enjoyed it

Robert: "

Question 68: He never lets anything

Mike:

B. w i l l be wearing C. wears

B. even though

A. Shall you make some coffee, please

Question 57: ' • Y o u ' l l recognize Jenny when you see her. She
A. w i l l wear

A. however

C. isn't


evolution in the 19"' century and of development in the 20"' century. Cultures
evolve or develop. They are not static. They have twists and turns. Styles change.
So do fashions, 'fhcre are cultural processes. What, for example, the word cultured
'Tieans has changed substantially since the study of classical (that is, Greek and
Roman) literature, philosophy, and history ceased in the 20"' century to be central

D . were
149


to school and university education. No single alternative locus enicrgcd, although
with computers has come electronic culture, affecting kinds of study, and most
recently digital culture. As cultures express themselves in new forms not
everything gets better or more civilized.
The multiplicity of meanings attached to the word made and will make it
difficult to define. There is no single, unproblematic definition, although many
attempts have been made to establish one. The only non-problematic definitions go
back to agricultural meaning (for example, cereal culture or strawberry culture) and
medical meaning (for example, bacterial culture or penicillin culture). Since in
anthropology and sociology we also acknowledge culture clashes, culture shock,
and counter-culture, the range of reference is extremely wide.
Question 71: According to the passage, the word culture
.
A. is related to the preparation and use of land for farming
B. develops from Greek and Roman literature and history

its

D. do not develop from the same meaning
Question 74: It can be inferred from the passage that since the 20"' century


.

, ,

A. agricultural and medical meanings
B. historical and figurative meanings



C. philosophical and historical meanings

3 i-..

D. sociological and anthropological meanings

; ^ •

Question 79: Which of the following is NOT true about the word culture?
in' j •

A. It is a word that cannot be defined.

oi» fin ,

C. comes from a source that has not been identified
D. derives from the same root as civilization does
Question 72: It is stated in paragraph 1 that a cultured person
.
j^^.

A. has a job related to cultivation
.
B. takes care of the soil and what grows on it
C. has knowledge of arts, literature, and music
D. does a job relevant to education
Question 73: The author remarks that culture and civilization are the two words
that
.
A. share the same word formation pattern
B. are both related to agriculture and cultivation
i C. have nearly the same meaning
.

A. schools and universities have not taught classical literature, philosophy, and
history
B. classical literature, philosophy, and history have been considered as core
•i
subjects
I C. classical literature, philosophy, and history have not been taught as
1
compulsory subjects
i D. all schools and universities have taught classical literature, philosophy, and
history
tr.
Question 75: The word "'attributes" in paragraph 1 most likely means
.
A. aspects
B.
fields
C. qualities

D. skills
Question 76: The word "static" in paragraph 2 could best be replaced by "
A. regular
B. balanced
C. unchanged
D. dense
150

estion 77: Which of the following is NOT stated in the passage?
A. Anthropology and sociology have tried to limit the references to culture.
B. Distinctions have been drawn between culture and civilization.
, ,
C. The word culture can be used to refer to a whole way of life of people. , ,
D. The use of the word culture has been changed since the 19"' century.
Question 78: It is difficult to give the definitions of the word culture EXCEPT for

B. Its use has been considerably changed.
C. It differs from the word civilization.
D. It evolves from agriculture.
Question 80: The passage mainly discusses

.

A. the distinction between culture and civilization
B. the figurative meanings of the word culture
C. the derivatives of the word culture

\K\j
•ir^


D. the multiplicity of meanings of the word culture

Question 1:

Huong dan lam bai chi tiet
grow (linking verb) + tinh tir = (dan dan) tra nen

S
'

C Dap an di'ing: grown fond of the surroundings (trcr nen yeu thich moi truong
xung quanh) = loved the surroundings.
A Phiiong an sai: planted many trees in the surroundings (trong nhieu cay a
moi truong xung quanh).
B Phirong an s a i : ( c 6 ma am, bj am anh - ngliTa tieu circ).
D Phuang an sai: possessed by/with (bj am anh - nghTa tieu eye).
Question!:
>:-^u.i.MU/
D Dap an dung: review = sir phe binh, bai phe binh mot cuon sach..); opinion =
Sir daiih gia.
///.V new work has enjoyed a very good review from critics and readers. (Tac pham
fioi cua ong ay nhan duoc sir danh gia rat tot tir cac nha phe binh va doc gia.)
A, B, C Phuang an sai: viewing (sir suy nghT v^ ai/cai gi theo mot each nhat

,, ;

Question 3:
B Dap an diing: universal (ph6 quat, pho bien, chung), shared (c6 chung cai gi).


i

151


.S'//c77 problems as haste and inexperience are a feature ofyouth.(^\\vsng van de
nliir su hap tap vii t h i l u kinh nghiem la diem dac trirng chung ciia thanh nien.)
A , C, D Phuong an sai: nuirked (bieu 16, bieu thj - khong dung lam b6 ngCr cho

C Dap an diing: access (to something) = ca hoi/ quyen sir dung... access to
critical services such as education, health care, and communication...
(ca hoi su'
dung cac dich vu then chot nhu giao due, cham siic y te va thong tin lien lac).
A , B , D Phuang an sai: challenge of something

danh tir); hidden (che giaii); separated (tach roi).

= thach thi'rc; key to

Question 4: K h i ket hop hai cau c6 cung chu tir, ta g i u lai menh de chinh va rut

= chia khoa, giai phap, bi quyet de dat cai g i ; advantage of something

gon menh dk phu. Menh d§ phu c6 the rut gon thanh:

Question 10:

-

Menli de phii mang nghla bj dong (The hook was published


while urban America

in the UKj ncMi ta

in the UK, the book has won a number of awards in recent

to

in rural areas than in urban areas. (Va trong khi

A , C, D Phuang an sai: because (bdi v i , do); when (khi); since (do, kk tir).

»-

Question 11:

(Xuat ban a Anii, quyen sach do danh dugc mot so ^;

D Dap an diing: pattern

giai tliuong a cac hoi clig sach trong khu vuc giin day.)
Question 5: Trong cum giai tir dugc rut gon tir menh de phu dung daii cau c6 diing
Dap an dimg B: it was seen—> we saw
During our tour of the refinery, we saw that both propane and gasoline

(= the regular way in which something happens or is

done): man, kieu. Tir pattern neu dac trung chung cho cac vimg nong thon MT.
Siiiall-.scale,


tinh tir so iiuu "our" ncn mtMih de chinh phai a dang chu dong voi chu tir "we".

were produced

look very different

upon these things, the challenges

cugc song a cac viing nong thon khac nhieu so v a i viing do thj.)

Dap an la A : Publishing in the U K - » Published in the U K
regional book fairs.

well-being

is equally dependent

viing do thi MT phu thugc dong deu vao cac yeu to nay thi cac thach thuc doi v a i

dinig cum qua khu phan tu.
Published

= thuan Igi.

B Dap an dung: lien tir while dugc dung de n6i hai sir viec doi lap nhau. And,

cum hien tai phan t u V - i n g (present participle phrase) mang ngliTa chu dong
cum qua khu phan tir V-ed/V3 (past particple phrase) mang nghTa bi dong


something

low-density settlement patterns make it more costly for

conmniriities

idit

and busine.'ises to provide critical services.

4i

lam cho cac cgng d6ng va doanh nghiep t6n kem trong viec cung cSp cac djch vu

(Kieu dinh cu thua that v o i quy mo nho

then chot.)

in large volumes. (Trong chuycn tham nha may Igc dau,

A , B , C Phuang an sai: style (= the particular way in which something is done):

ciiung tui tluiy proban va xang dau duoc tinh che v o i dung lugng Ion.)
Question 6: Ta phai tiieo cau triic song hanh (parallel structure): cac danh tir lam

phong each, kieu, loi - diing de chi tirng truong hop nhat dinh chu khong mang

b6 ngu cho dong tir "be"' dugc noi voi nhau bang lien tir dang lap "and".

ti'nh khai quat; tool (dung cu); means (phuang tien, each thuc).


Dap an dung D: you have good health —>• good health
The fust

important

requirements

for you to become a mountain

climber

tra thanh nha leo nui la ban phai c6 niem dam me manh liet va sue khoe tot.)
Question 7: V o i cau triic hardly...when (vira m a i . . . t h i ) , ta dung thi Qua khii
hoan thanh (Past Perfect) cho hanh dong thir nhat hoan tat trudc hanh dong kia.

Declining Jobs and income in the natural resource-based

a living. (Su giam siit viec lam va thu nhap a cac nganh cong nghiep dua vao tai
nguyen ma nhieu vimg nong nghiep le thugc do bugc cong nhan a cac nganh cong
nghiep nay tim each moi de kiem song.)

Hardly did he enter the room when all the lights went out. (Anh ay moi viia vao
phong thi tat ca bong den tat.)
(ncMi kinh tc)-

economics (kinh te hgc).
the relationship

of economy and history at our university developed a new theory

between historical

events and fnancial

crises.

kinii t6 Iigc va Hch sir a truong dai hoc chimg ta da phat trien mot ly tiuiyct moi v-"
Question 9: Ciiu nay hoi ve kha nang ket hop tir (word collocation).

A , B , D Phuong an sai: offer (someone) something = cung cap. tao ca hoi; turn
(linking verb) + tinh tii' = chuyen, lam cho; make someone do something = hk, bk bugc.
Question 13:

la

B, C, D Phuong an sai: abroad (trang tir) = a nuac ngoai; lateral = a ben; rural
^ tliiigc ve nong thon (khong phii hgp voi cac nganh cong nghiep a nong thon gap
•^lifing doi thu canh tranh tir nai khac).

«y«\v.s\\ti

Question 14:
D Dap an diing: affect (dong tir) = anh huong/ tac dong den ai/ cai gi. Finally,
changes in the availability and use of natural

152

i)

A Dap an dimg: foreign competitors (doi thii canh tranh nuac ngoai), foreign

h'nh tir b6 ngu cho danh tir competitors.

ol

( M o t vj giao su

moi quan he giua cac sir kien lich sir va cac cugc khinig hoang tai chinh.)

industries that many

rural areas depeiul on force workers in those industries to find new ways to make

Dap an dimg A : did he enter —> had he entered

Question 8: Cau nay hoi ve tir vung, ta phai phan biet: economy

.j ,

C Dap an dung: force someone to do something (huge ai lam coi go).

are

your strou},' passion and you have },'ood health. (Yen cau quan trong dau tien dc

A professor

^

Question 12:


f


×